2008AMC1012A-TM-problems

合集下载

2008 AMC 12A Problems(答案)

2008 AMC 12A Problems(答案)

2008 AMC 12A ProblemsProblem 1A bakery owner turns on his doughnut machine at . At themachine has completed one third of the day's job. At what time will the doughnut machine complete the job?SolutionThe machine completes one-third of the job in hours. Thus,the entire job is completed in hours.Since the machine was started at , the job will be finished hours later, at. The answer is .Problem 2What is the reciprocal of ?Solution.Problem 3Suppose that of bananas are worth as much as oranges. How many orangesare worth as much as of bananas?SolutionIf , then.Problem 4Which of the following is equal to the productSolutionSolution 1.Solution 2Notice that everything cancels out except for in the numerator and in thedenominator.Thus, the product is , and the answer is .Problem 5Suppose that is an integer. Which of the following statements must be true about ?SolutionFor to be an integer, mustbe even, but not necessarily divisible by . Thus, the answer is .Problem 6Heather compares the price of a new computer at two different stores. Storeoffers off the sticker price followed by a rebate, and store offers offthe same sticker price with no rebate. Heather saves by buying the computer at store instead of store . What is the sticker price of the computer, in dollars?SolutionSolution 1Let the sticker price be .The price of the computer is at store , and at store .Heather saves at store , so .Solving, we find , and the thus answer is .Solution 2The in store is better than the additional off at store .Thus the off is equal to , and therefore the sticker price is .Problem 7While Steve and LeRoy are fishing 1 mile from shore, their boat springs a leak, and water comes in at a constant rate of 10 gallons per minute. The boat will sink if it takes in more than 30 gallons of water. Steve starts rowing toward the shore at a constant rate of 4 miles per hour while LeRoy bails water out of the boat. What is the slowest rate, in gallons per minute, at which LeRoy can bail if they are to reach the shore without sinking?SolutionIt will take of an hour orminutes to get to shore.Since only gallons of water can enter the boat, only net gallons can enterthe boat per minute. Sincegallons of water enter the boat each minute, LeRoy must bailgallons per minute.Problem 8What is the volume of a cube whose surface area is twice that of a cube with volume 1?SolutionA cube with volume has a side of lengthand thus a surface area of.A cube whose surface area is has a side of length and avolume of .Problem 9Older television screens have an aspect ratio of . That is, the ratio of the widthto the height is . The aspect ratio of many movies is not , so they aresometimes shown on a television screen by "letterboxing" - darkening strips of equal height at the top and bottom of the screen, as shown. Suppose a movie has an aspect ratio of and is shown on an older television screen with a -inchdiagonal. What is the height, in inches, of each darkened strip?SolutionLet the width and height of the screen be and respectively, and let the widthand height of the movie be and respectively.By the Pythagorean Theorem, the diagonal is . So.Since the movie and the screen have the same width, .Thus, the height of each strip is .Problem 10Doug can paint a room in hours. Dave can paint the same room in hours. Dougand Dave paint the room together and take a one-hour break for lunch. Let be the total time, in hours, required for them to complete the job working together, including lunch. Which of the following equations is satisfied by ?SolutionDoug can paint of a room per hour, Dave can paint of a room in an hour, and the time they spend working together is .Since rate times time gives output,Problem 11Three cubes are each formed from the pattern shown. They are then stacked on a table one on top of another so that the visible numbers have the greatest possiblesum. What is that sum?SolutionTo maximize the sum of the faces that are showing, we can minimize the sum ofthe numbers of the faces that are not showing.The bottom cubes each have a pair of opposite faces that are covered up. Whenthe cube is folded, ; ; and are opposite pairs. Clearlyhas the smallest sum.The top cube has 1 number that is not showing. The smallest number on a face is .So, the minimum sum of the unexposed faces is . Since the sum ofthe numbers on all the cubes is , the maximumpossible sum of visible numbers is .Problem 12A function has domain and range. (The notation denotes.) What are the domain and range, respectively, of the functiondefined by ?Solutionis defined if is defined. Thus the domain is all.Since , . Thusis the range of .Thus the answer is .Problem 13Points and lie on a circle centered at , and . A second circle isinternally tangent to the first and tangent to both and . What is the ratio of the area of the smaller circle to that of the larger circle?SolutionLet be the center of the small circle with radius , and let be the point wherethe small circle is tangent to . Also, let be the point where the small circle istangent to the big circle with radius .Then is a right triangle, and a triangle at that. Therefore,.Since , we have , or, or .Then the ratio of areas will be squared, or .Problem 14What is the area of the region defined by the inequality?Area is invariant under translation, so after translating left and up units, wehave the inequalitywhich forms a diamond centered at the origin and vertices at .Thus the diagonals are of length and . Using the formula , theanswer is .Problem 15Let . What is the units digit of ?Solution.So, . Since is a multiple of four and the unitsdigit of powers of two repeat in cycles of four, .Therefore, . So the units digit is .Problem 16The numbers , , and are the first three terms of anarithmetic sequence, and the term of the sequence is . What is ?Solution 1Let and .The first three terms of the arithmetic sequence are , , and, and the term is .Thus, .Since the first three terms in the sequence are , , and , the th termis .Thus the term is .Solution 2If , , and are in arithmetic progression, then ,, and are in geometric progression. Therefore,Therefore, , , therefore the 12th term in the sequence isProblem 17Let be a sequence determined by the rule if is evenand if is odd. For how many positive integers is it true that is less than each of , , and ?SolutionAll positive integers can be expressed as , , , or , whereis a nonnegative integer.▪ If , then .▪ If , then ,, and.▪If, then.▪ If , then,, and.Since , every positive integerwill satisfy.Since one fourth of the positive integers can be expressed as,where is a nonnegative integer, the answer is.Problem 18Triangle, with sides of length , , and , has one vertex on the positive-axis, one on the positive -axis, and one on the positive -axis. Let be the origin .What is the volume of tetrahedron?SolutionWithout loss of generality, let be on the axis, be on the axis, and be onthe axis, and let have respective lengths of 5, 6, and 7. Letdenote the lengths of segments respectively. Then by thePythagorean Theorem, so ; similarly,and . Since , , and are mutually perpendicular, thetetrahedron's volume is which is answer choice C.Problem 19In the expansion of what isthe coefficient of ?SolutionLet and . We areexpanding .Since there are terms in , there are ways to choose one term fromeach . The product of the selected terms is for some integer between andinclusive. For each , there is one and only one in . Since there isonly one way to choose one term from each to get a product of , there areways to choose one term from each and one term from to get aproduct of . Thus the coefficient of the term is .Problem 20Triangle has , , and . Point is on , andbisects the right angle. The inscribed circles of and have radiiand , respectively. What is ?SolutionBy the Angle Bisector Theorem, By Law of Sines on, Since the areaof a triangle satisfies , where the inradius and the semiperimeter,we have Since and share the altitude (to ),their areas are the ratio of their bases, or The semiperimetersare and . Thus,Problem 21A permutation of is heavy-tailed if. What is the number of heavy-tailed permutations?SolutionThere are total permutations.For every permutation such that , there isexactly one permutation such that . Thus it suffices to count thepermutations such that ., , and are the only combinations of numbers that can satisfy .There are combinations of numbers, possibilities of which side of the equation isand which side is , and possibilities for rearrangingand . Thus, there are permutations such that.Thus, the number of heavy-tailed permutations is .Problem 22A round table has radius . Six rectangular place mats are placed on the table. Eachplace mat has width and length as shown. They are positioned so that each mathas two corners on the edge of the table, these two corners being end points of the same side of length . Further, the mats are positioned so that the inner corners each touch an inner corner of an adjacent mat. What is ?SolutionSolution 1 (trigonometry)Let one of the mats be , and the center be as shown:Since there are mats, is equilateral. So, . Also,.By the Law of Cosines:.Since must be positive, .Solution 2 (without trigonometry)Draw and as in the diagram. Draw the altitude from to and call theintersectionAs proved in the first solution, . That makes atriangle, so andSince is a right triangle,Solving for givesProblem 23The solutions of the equation are the vertices of a convex polygon in the complex plane. What is the area of the polygon?SolutionLooking at the coefficients, we are immediately reminded of the binomial expansionof .Modifying this slightly, we can write the given equation as:We can apply a translation of and a rotation of(both operations preserve area) to simplify the problem:Because the roots of this equation are created by rotating radians successively about the origin, the quadrilateral is a square.We know that half the diagonal length of the square isTherefore, the area of the square isProblem 24Triangle has and . Point is the midpoint of . What isthe largest possible value of ?SolutionLet . Then , and sinceand , we haveWith calculus, taking the derivative and setting equal to zero will give the maximum value of . Otherwise, we can apply AM-GM:Thus, the maximum is at .Problem 25A sequence , , , of points in the coordinate plane satisfiesfor .Suppose that . What is ?SolutionThis sequence can also be expressed using matrix multiplication as follows:.Thus, is formed by rotating counter-clockwise about the originbyand dilating the point's position with respect to the origin by a factor of .So, starting with and performing the above operationstimes inreverse yields.Rotating clockwise by yields . A dilation by a factor ofyields the point .Therefore, .。

2010 AMC 10A 试题及答案解析

2010 AMC 10A 试题及答案解析

2010 AMC 10A problems and solutions.The test was held on February 8, 2010. The first link contains the full set of test problems. The rest contain each individual problem and its solution.Problem 1Mary’s top book shelf holds five books with the follow ing widths, incentimeters: , , , , and . What is the average book width, in centimeters?SolutionTo find the average, we add up the widths , , , , and , to get a total sum of . Since there are books, the average book width isThe answer is .Problem 2Four identical squares and one rectangle are placed together to form one large square as shown. The length of the rectangle is how many times as large as its width?SolutionLet the length of the small square be , intuitively, the length of the big square is . It can be seen that the width of the rectangle is .Thus, the length of the rectangle is times large as the width. The answer is .Problem 3Tyrone had marbles and Eric had marbles. Tyrone then gave some of his marbles to Eric so that Tyrone ended with twice as many marbles as Eric. How many marbles did Tyrone give to Eric?SolutionLet be the number of marbles Tyrone gave to Eric. Then,. Solving for yields and . The answer is .Problem 4A book that is to be recorded onto compact discs takes minutes to read aloud. Each disc can hold up to minutes of reading. Assume that the smallest possible number of discs is used and that each disc contains the same length of reading. How many minutes of reading will each disc contain?SolutionAssuming that there were fractions of compact discs, it would take CDs to have equal reading time. However, since the number of discs can only be a whole number, there are at least 8 CDs, in which case it would have minutes on each of the 8 discs. The answer is .Problem 5The area of a circle whose circumference is is . What is the value of ?SolutionIf the circumference of a circle is , the radius would be . Since the area of a circle is , the area is . The answer is . Problem 6For positive numbers and the operation is defined asWhat is ?Solution. Then, is The answer isProblem 7Crystal has a running course marked out for her daily run. She starts this run by heading due north for one mile. She then runs northeast for one mile, then southeast for one mile. The last portion of her run takes her on a straight line back to where she started. How far, in miles is this last portion of her run?SolutionCrystal first runs North for one mile. Changing directions, she runs Northeast for another mile. The angle difference between North and Northeast is 45 degrees. She then switches directions to Southeast, meaning a 90 degree angle change. The distance now from travelling North for one mile, and her current destination is miles, because it is the hypotenuse of a 45-45-90 triangle with side length one (mile). Therefore, Crystal's distance from her starting position, x, is equal to , which is equal to . The answer isTony works hours a day and is paid $per hour for each full year of his age. During a six month period Tony worked days and earned $. How old was Tony at the end of the six month period?SolutionTony worked hours a day and is paid dollars per hour for each full year of his age. This basically says that he gets a dollar for each year of his age. So if he is years old, he gets dollars a day. We also know that he worked days and earned dollars. If he was years old at the beginning of his working period, he would have earned dollars. If he was years old at the beginning of his working period, he would have earned dollars. Because he earned dollars, we know that he was for some period of time, but not the whole time, because then the money earned would be greater than or equal to . This is why he was when he began, but turned sometime in the middle and earned dollars in total. So the answer is .The answer is . We could find out for how long he was and . . Then isand we know that he was for days, and for days. Thus, the answer is .Problem 9A palindrome, such as , is a number that remains the same when its digits are reversed. The numbers and are three-digit and four-digit palindromes, respectively. What is the sum of the digits of ?Solutionis at most , so is at most . The minimum value ofis . However, the only palindrome between and is , which means that must be .It follows that is , so the sum of the digits is .Marvin had a birthday on Tuesday, May 27 in the leap year . In what year will his birthday next fall on a Saturday?Solution(E) 2017There are 365 days in a non-leap year. There are 7 days in a week. Since 365 = 52 * 7 + 1 (or 365 is congruent to 1 mod 7), the same date (after February) moves "forward" one day in the subsequent year, if that year is not a leap year.For example: 5/27/08 Tue 5/27/09 WedHowever, a leap year has 366 days, and 366 = 52 * 7 + 2. So the same date (after February) moves "forward" two days in the subsequent year, if that year is a leap year.For example: 5/27/11 Fri 5/27/12 SunYou can keep count forward to find that the first time this date falls on a Saturday is in 2017:5/27/13 Mon 5/27/14 Tue 5/27/15 Wed 5/27/16 Fri 5/27/17 Sat Problem 11The length of the interval of solutions of the inequality is . What is ?SolutionSince we are given the range of the solutions, we must re-write the inequalities so that we have in terms of and .Subtract from all of the quantities:Divide all of the quantities by .Since we have the range of the solutions, we can make them equal to .Multiply both sides by 2.Re-write without using parentheses.Simplify.We need to find for the problem, so the answer isProblem 12Logan is constructing a scaled model of his town. The city's water tower stands 40 meters high, and the top portion is a sphere that holds 100,000 liters of water. Logan's miniature water tower holds 0.1 liters. How tall, in meters, should Logan make his tower?SolutionThe water tower holds times more water than Logan's miniature. Therefore, Logan should make his towertimes shorter than the actual tower. This ismeters high, or choice .Problem 13Angelina drove at an average rate of kph and then stopped minutes for gas. After the stop, she drove at an average rate of kph. Altogether she drove km in a total trip time of hours including the stop. Which equation could be used to solve for the time in hours that she drove before her stop?SolutionThe answer is ()because she drove at kmh for hours (the amount of time before the stop), and 100 kmh for because she wasn't driving for minutes, or hours. Multiplying by gives the total distance, which is kms. Therefore, the answer isProblem 14Triangle has . Let and be on and , respectively, such that . Let be the intersection of segments and , and suppose that is equilateral. What is ?SolutionLet .Since ,Problem 15In a magical swamp there are two species of talking amphibians: toads, whose statements are always true, and frogs, whose statements are always false. Four amphibians, Brian, Chris, LeRoy, and Mike live together in this swamp, and they make the following statements.Brian: "Mike and I are different species."Chris: "LeRoy is a frog."LeRoy: "Chris is a frog."Mike: "Of the four of us, at least two are toads."How many of these amphibians are frogs?SolutionSolution 1We can begin by first looking at Chris and LeRoy.Suppose Chris and LeRoy are the same species. If Chris is a toad, then what he says is true, so LeRoy is a frog. However, if LeRoy is a frog, then he is lying, but clearly Chris is not a frog, and we have a contradiction. The same applies if Chris is a frog.Clearly, Chris and LeRoy are different species, and so we have at least frog out of the two of them.Now suppose Mike is a toad. Then what he says is true because we already have toads. However, if Brian is a frog, then he is lying, yet his statement is true, a contradiction. If Brian is a toad, then what he says is true, but once again it conflicts with his statement, resulting in contradiction.Therefore, Mike must be a frog. His statement must be false, which means that there is at most toad. Since either Chris or LeRoy is already a toad, Brain must be a frog. We can also verify that his statement is indeed false.Both Mike and Brian are frogs, and one of either Chris or LeRoy is a frog, so we have frogs total.Solution 2Start with Brian. If he is a toad, he tells the truth, hence Mike is a frog. If Brian is a frog, he lies, hence Mike is a frog, too. Thus Mike must be a frog.As Mike is a frog, his statement is false, hence there is at most one toad.As there is at most one toad, at least one of Chris and LeRoy is a frog. But then the other one tells the truth, and therefore is a toad. Hence we must have one toad and three frogs.Problem 16Nondegenerate has integer side lengths, is an angle bisector, , and . What is the smallest possible value of the perimeter?SolutionBy the Angle Bisector Theorem, we know that . If we use the lowest possible integer values for AB and BC (the measures of AD and DC, respectively), then , contradicting the Triangle Inequality. If we use the next lowest values (and ), the Triangle Inequality is satisfied. Therefore, our answer is , or choice .Problem 17A solid cube has side length inches. A -inch by -inch square hole is cut into the center of each face. The edges of each cut are parallel to the edges of the cube, and each hole goes all the way through the cube. What is the volume, in cubic inches, of the remaining solid?SolutionSolution 1Imagine making the cuts one at a time. The first cut removes a box . The second cut removes two boxes, each of dimensions, and the third cut does the same as the second cut, on the last two faces. Hence the total volume of all cuts is .Therefore the volume of the rest of the cube is.Solution 2We can use Principle of Inclusion-Exclusion to find the final volume of the cube.There are 3 "cuts" through the cube that go from one end to the other. Each of these "cuts" has cubic inches. However, we can not just sum their volumes, as the central cube is included in each of these three cuts. To get the correct result, we can take the sum of the volumes of the three cuts, and subtract the volume of the central cube twice.Hence the total volume of the cuts is.Therefore the volume of the rest of the cube is.Solution 3We can visualize the final figure and see a cubic frame. We can find the volume of the figure by adding up the volumes of the edges and corners.Each edge can be seen as a box, and each corner can be seen as a box..Problem 18Bernardo randomly picks 3 distinct numbers from the setand arranges them in descending order to form a 3-digit number. Silvia randomly picks 3 distinct numbers from the set and also arranges them in descending order to form a 3-digit number. What is the probability that Bernardo's number is larger than Silvia's number?SolutionWe can solve this by breaking the problem down into cases and adding up the probabilities.Case : Bernardo picks . If Bernardo picks a then it is guaranteed that his number will be larger than Silvia's. The probability that he will pick a is .Case : Bernardo does not pick . Since the chance of Bernardo picking is , the probability of not picking is .If Bernardo does not pick 9, then he can pick any number from to . Since Bernardo is picking from the same set of numbers as Silvia, the probability that Bernardo's number is larger is equal to the probability that Silvia's number is larger.Ignoring the for now, the probability that they will pick the same number is the number of ways to pick Bernardo's 3 numbers divided by the number of ways to pick any 3 numbers.We get this probability to beProbability of Bernardo's number being greater isFactoring the fact that Bernardo could've picked a but didn't:Adding up the two cases we getProblem 19Equiangular hexagon has side lengthsand . The area of is of the area of the hexagon. What is the sum of all possible values of ?SolutionSolution 1It is clear that is an equilateral triangle. From the Law of Cosines, we get that . Therefore, the area of is .If we extend , and so that and meet at , and meet at , and and meet at , we find that hexagon is formed by taking equilateral triangle of side length and removing three equilateral triangles, , and , of side length . The area of is therefore.Based on the initial conditions,Simplifying this gives us . By Vieta's Formulas we know that the sum of the possible value of is .Solution 2As above, we find that the area of is .We also find by the sine triangle area formula that, and thusThis simplifies to.Problem 20A fly trapped inside a cubical box with side length meter decides to relieve its boredom by visiting each corner of the box. It will begin and end in the same corner and visit each of the other corners exactly once. To get from a corner to any other corner, it will either fly or crawl in a straight line. What is the maximum possible length, in meters, of its path?SolutionThe distance of an interior diagonal in this cube is and the distance of a diagonal on one of the square faces is . It would not make sense if the fly traveled an interior diagonal twice in a row, as it would return to the point it just came from, so at most the final sum can only have 4 as the coefficient of . The other 4 paths taken can be across a diagonal on one of the faces, so the maximum distance traveled is.Problem 21The polynomial has three positive integer zeros. What is the smallest possible value of ?SolutionBy Vieta's Formulas, we know that is the sum of the three roots of the polynomial . Also, 2010 factors into. But, since there are only three roots to the polynomial, two of the four prime factors must be multiplied so that we are left with three roots. To minimize , and should be multiplied, which means will be and the answer is .Problem 22Eight points are chosen on a circle, and chords are drawn connecting every pair of points. No three chords intersect in a single point insidethe circle. How many triangles with all three vertices in the interior of the circle are created?SolutionTo choose a chord, we know that two points must be chosen. This implies that for three chords to create a triangle and not intersect at a single point, six points need to be chosen. Therefore, the answer is which is equivalent to 28,Problem 23Each of 2010 boxes in a line contains a single red marble, and for , the box in the position also contains white marbles. Isabella begins at the first box and successively draws a single marble at random from each box, in order. She stops when she first draws a red marble. Let be the probability that Isabella stops afterdrawing exactly marbles. What is the smallest value of for which ?SolutionThe probability of drawing a white marble from box is . Theprobability of drawing a red marble from box is .The probability of drawing a red marble at box is thereforeIt is then easy to see that the lowest integer value of that satisfies the inequality is .Problem 24The number obtained from the last two nonzero digits of is equal to . What is ?SolutionWe will use the fact that for any integer ,First, we find that the number of factors of in is equal to. Let . The we want is therefore the last two digits of , or . Since there is clearly an excess of factors of 2, we know that , so it remains to find .If we divide by by taking out all the factors of in , we canwrite as where where every multiple of 5 is replaced by the number with all its factors of 5 removed. Specifically, every number in the form is replaced by , and every number in the form is replaced by .The number can be grouped as follows:Using the identity at the beginning of the solution, we can reducetoUsing the fact that (or simply the fact that if you have your powers of 2 memorized), we can deduce that . Therefore.Finally, combining with the fact that yields.Problem 25Jim starts with a positive integer and creates a sequence of numbers. Each successive number is obtained by subtracting the largest possible integer square less than or equal to the current number until zero is reached. For example, if Jim starts with , then his sequence contains numbers:Let be the smallest number for which Jim’s sequence has numbers. What is the units digit of ?SolutionWe can find the answer by working backwards. We begin with on the bottom row, then the goes to the right of the equal's sign in the row above. We find the smallest value for whichand , which is .We repeat the same procedure except with for the next row and for the row after that. However, at the fourth row, wesee that solving yields , in which case it would be incorrect since is not the greatest perfect square less than or equal to . So we make it a and solve . We continue on using this same method where we increase the perfect square until can be made bigger than it. When we repeat this until we have rows, we get:Hence the solution is the last digit of , which is .。

2008-AMC10-Bsolutions

2008-AMC10-Bsolutions

T he MATheMATICAL ASSOCIATION Of AMeRICAAmerican Mathematics Competitions9th Annual American Mathematics Contest 10AMC 10Contest BSolutions PamphletWednesday, FEBRUARY 27, 2008This Pamphlet gives at least one solution for each problem on this year’s contest and shows that all problems can be solved without the use of a calculator. When more than one solution is provided, this is done to illustrate a significant contrast in methods, e.g., algebraic vs geometric, computational vs conceptual, elementary vs advanced. These solu-tions are by no means the only ones possible, nor are they superior to others the reader may devise.We hope that teachers will inform their students about these solutions, both as illustrations of the kinds of ingenuity needed to solve nonroutine problems and as examples of good mathematical exposition. However,the publication, reproduction or communication of the problems or solutions of the AMC 10 during the period when students are eligible to participate seriously jeopardizes the integrity of the results. Dissemination via copier, telephone, e-mail, WorldWide Web or media of any type during this period is a violation of the competition rules.After the contest period, permission to make copies of problems in paper or electronic form including posting on web-pages for educational use is granted without fee provided that copies are not made or distributed for profit or commercial advantage and that copies bear the copyright notice.Correspondence about the problems/solutions for this AMC 10 and orders for any publications should be addressed to:American Mathematics CompetitionsUniversity of Nebraska, P.O. Box 81606, Lincoln, NE 68501-1606Phone: 402-472-2257; Fax: 402-472-6087; email: amcinfo@ The problems and solutions for this AMC 10 were prepared by the MAA’s Committee on the AMC 10 and AMC 12 under the direction of AMC 10 Subcommittee Chair:Chair: LeRoy Wenstrom, Columbus, MSCopyright © 2008, The Mathematical Association of America1.Answer (E):The number of points could be any integer between 5·2=10and 5·3=15,inclusive.The number of possibilities is 15−10+1=6.123489101115161718222324252.Answer (B):The two sums are 1+10+17+22=50and4+9+16+25=54,so the positive difference between the sumsis 54−50=4.Query:If a different 4×4block of dates had been chosen,theanswer would be unchanged.Why?3.Answer (D):The properties of exponents imply that3x √x = x ·x 12 13= x 32 13=x 12.4.Answer (C):A single player can receive the largest possible salary only when the other 20players on the team are each receiving the minimum salary of $15,000.Thus the maximum salary for any player is $700,000−20·$15,000=$400,000.5.Answer (A):Note that (y −x )2=(x −y )2,so(x −y )2$(y −x )2=(x −y )2$(x −y )2= (x −y )2−(x −y )2 2=02=0.6.Answer (C):Because AB +BD =AD and AB =4BD ,it follows thatBD =15·AD .By similar reasoning,CD =110·AD .ThusBC =BD −CD =15·AD −110·AD =110·AD.7.Answer (C):The side length of the large triangle is 10times the side length of each small triangle,so the area of the large triangle is 102=100times the area of each small triangle.8.Answer (C):The total cost of the carnations must be an even number of dollars.The total number of dollars spent is the even number 50,so the number of roses purchased must also be even.In addition,the number of roses purchased cannot exceed 503.Therefore the number of roses purchased must be one of theeven integers between 0and 16,inclusive.This gives 9possibilities for the number of roses purchased,and consequently 9possibilities for the number of bouquets.9.Answer (A):The quadratic formula implies that the two solutions arex 1=2a +√4a 2−4ab 2a and x 2=2a −√4a 2−4ab 2a,so the average is 12(x 1+x 2)=122a 2a +2a 2a =1.OR The sum of the solutions of a quadratic equation is the negative of the coefficient of the linear term divided by the coefficient of the quadratic term.In this case the sum of the solution is −(−2a )a=2.Hence the average of the solutions is1.10.Answer (A):Let O be the center of the circle,and let D be the intersection of OC and AB .Be-cause OC bisects minor arc AB ,OD is a perpen-dicular bisector of chord AB .Hence AD =3,andapplying the Pythagorean Theorem to ADO yieldsOD =√52−33=4.Therefore DC =1,and apply-ing the Pythagorean Theorem to ADC yields AC =√32+12=√10.11.Answer (B):Note that u 5=2u 4+9and 128=u 6=2u 5+u 4=5u 4+18.Thus u 4=22,and it follows that u 5=2·22+9=53.12.Answer (A):During the year Pete takes44×105+5×104=44.5×105steps.At 1800steps per mile,the number of miles Pete walks is44.5×10518×10=44.518×103≈2.5×103=2500.13.Answer (B):Because the mean of the first n terms is n ,their sum is n 2.Therefore the n th term is n 2−(n −1)2=2n −1,and the 2008th term is 2·2008−1=4015.14.Answer (B):Because OAB is a 30–60–90◦triangle,we have BA =5√33.Let A and B be the images of A and B ,respectively,under the rotation.ThenB =(0,5),B A is horizontal,and B A =BA=5√33.Hence A is in thesecond quadrant andA =−53√3,5.15.Answer(A):By the Pythagorean Theorem we have a2+b2=(b+1)2,soa2=(b+1)2−b2=2b+1.Because b is an integer with b<100,a2is an odd perfect square between1and 201,and there are six of these,namely,9,25,49,81,121,and169.Hence a must be3,5,7,9,11,or13,and there are6triangles that satisfy the given conditions.16.Answer(A):If one die is rolled,3of the6possible numbers are odd.If twodice are rolled,18of the36possible outcomes have odd sums.In each of thesecases,the probability of an odd sum is12.If no die is rolled,the sum is0,whichis not odd.The probability that no die is rolled is equal to the probability thatboth coin tosses are tails,which is(12)2=14.Thus the requested probability is1−14·12=38.17.Answer(B):The responses on these three occasions,in order,must beYNN,NYN,or NNY,where Y indicates approval and N indicates disapproval.The probability of each of these is(0.7)(0.3)(0.3)=0.063,so the requested probability is3(0.063)=0.189.18.Answer(B):Let n be the number of bricks in the chimney.Then thenumber of bricks per hour Brenda and Brandon can lay working alone is n9andn 10,respectively.Working together they can lay(n9+n10−10)bricks in an hour,or5 n9+n10−10bricks in5hours to complete the chimney.Thus5 n9+n10−10=n,and the number of bricks in the chimney is n=900.ORSuppose that Brenda can lay x bricks in an hour and Brandon can lay y bricks in an hour.Then the number of bricks in the chimney can be expressed as9x,10y ,or 5(x +y −10).The equality of these expressions leads to the system of equations4x −5y =−50−5x +5y =−50.It follows that x =100,so the number of bricks in the chimney is 9x =900.19.Answer (E):The portion of each end of the tank thatis under water is a circular sector with two right trianglesremoved as shown.The hypotenuse of each triangle is 4,and the vertical leg is 2,so each is a 30–60–90◦triangle.Therefore the sector has a central angle of 120◦,and thearea of the sector is 120360·π(4)2=163π.The area of each triangle is 12(2) 2√3 ,so the portion of each end that is underwater has area 163π−4√3.The length of the cylinder is 9,so the volume of the water is 9 163π−4√3 =48π−36√3.20.Answer (B):Of the 36possible outcomes,the four pairs (1,4),(2,3),(2,3),and (4,1)yield a sum of 5.The six pairs (1,6),(2,5),(2,5),(3,4),(3,4),and (4,3)yield a sum of 7.The four pairs (1,8),(3,6),(3,6),and (4,5)yield a sum of 9.Thus the probability of getting a sum of 5,7,or 9is (4+6+4)/36=7/18.Note:The dice described here are known as Sicherman dice.The probability of obtaining each sum between 2and 12is the same as that on a pair of standard dice.21.Answer (C):Let the women be seated first.The first woman may sit inany of the 10chairs.Because men and women must alternate,the number of choices for the remaining women is 4,3,2,and 1.Thus the number of possible seating arrangements for the women is 10·4!=240.Without loss of generality,suppose that a woman sits in chair 1.Then this woman’s spouse must sit in chair 4or chair 8.If he sits in chair 4then the women sitting in chairs 7,3,9,and 5must have their spouses sitting in chairs 10,6,2,and 8,respectively.If he sits in chair 8then the women sitting in chairs 5,9,3,and 7must have their spouses sitting in chairs 2,6,10,and 4,respectively.So for each possible seating arrangement for the women there are two arrangements for the men.Hence,there are 2·240=480possible seating arrangements.22.Answer (C):There are 6!/(3!2!1!)=60distinguishable orders of the beadson the line.To meet the required condition,the red beads must be placed inone of four configurations:positions 1,3,and 5,positions 2,4,and 6,positions 1,3,and 6,or positions 1,4,and 6.In the first two cases,the blue bead can be placed in any of the three remaining positions.In the last two cases,the blue bead can be placed in either of the two adjacent remaining positions.In each case,the placement of the white beads is then determined.Hence there are 2·3+2·2=10orders that meet the required condition,and the requestedprobability is 1060=16.23.Answer (B):Because the area of the border is half the area of the floor,thesame is true of the painted rectangle.The painted rectangle measures a −2by b −2feet.Hence ab =2(a −2)(b −2),from which 0=ab −4a −4b +8.Add 8to each side of the equation to produce8=ab −4a −4b +16=(a −4)(b −4).Because the only integer factorizations of 8are8=1·8=2·4=(−4)·(−2)=(−8)·(−1),and because b >a >0,the only possible ordered pairs satisfying this equation for (a −4,b −4)are (1,8)and (2,4).Hence (a,b )must be one of the two ordered pairs (5,12),or (6,8).A B CD M AB C D O 24.Answer (C):Let M be on the same side of line BC asA ,such that BMC is equilateral.Then ABM andMCD are isosceles with ∠ABM =10◦and ∠MCD =110◦.Hence ∠AMB =85◦and ∠CMD =35◦.There-fore∠AMD =360◦−∠AMB −∠BMC −∠CMD=360◦−85◦−60◦−35◦=180◦.It follows that M lies on AD and ∠BAD =∠BAM =85◦.ORLet ABO be equilateral as shown.Then∠OBC =∠ABC −∠ABO =70◦−60◦=10◦.Because ∠BCD =170◦and OB =BC =CD ,the quadrilateral BCDO is a parallelogram.ThusOD=BC=AO and AOD is isosceles.Letα=∠ODA=∠OAD.The sum of the interior angles of ABCD is360◦,so we have360=(α+60)+70+170+(α+10)andα=25.Thus∠DAB=60+α=85◦.25.Answer(B):Number the pails consecutively so that Michael is presently atpail0and the garbage truck is at pail1.Michael takes200/5=40seconds to walk between pails,so for n≥0he passes pail n after40n seconds.The truck takes20seconds to travel between pails and stops for30seconds at each pail.Thus for n≥1it leaves pail n after50(n−1)seconds,and for n≥2it arrives at pail n after50(n−1)−30seconds.Michael will meet the truck at pail n if and only if50(n−1)−30≤40n≤50(n−1)or,equivalently,5≤n≤8.50time (sec)distance (ft)200truckMichaelHence Michaelfirst meets the truck at pail5after200seconds,just as the truck leaves the pail.He passes the truck at pail6after240seconds and at pail7 after280seconds.Finally,Michael meets the truck just as it arrives at pail8 after320seconds.These conditions imply that the truck is ahead of Michael between pails5and6and that Michael is ahead of the truck between pails7 and8.However,the truck must pass Michael at some point between pails6and 7,so they meet a total offive times.TheAmerican Mathematics Competitionsare Sponsored byThe Mathematical Association of AmericaThe Akamai FoundationContributorsAmerican Mathematical Association of Two Year CollegesAmerican Mathematical SocietyAmerican Society of Pension ActuariesAmerican Statistical AssociationArt of Problem SolvingAwesome MathCanada/USA MathcampCanada/USA MathpathCasualty Actuarial SocietyClay Mathematics InstituteIDEA MathInstitute for Operations Research and the Management SciencesL. G. Balfour CompanyMu Alpha ThetaNational Assessment & TestingNational Council of Teachers of MathematicsPedagoguery Software Inc.Pi Mu EpsilonSociety of ActuariesU.S.A. Math Talent SearchW. H. Freeman and CompanyWolfram Research Inc.。

AMC10的真题答案及中文翻译

AMC10的真题答案及中文翻译

AMC10的真题答案及中文翻译AMC10的真题及中文翻译1、One ticket to a show costs $20 at full price. Susan buys 4 tickets using a coupon that gives her a 25% discount. Pam buys 5 tickets using a coupon that gives her a 30% discount. How many more dollars does Pam pay than Susan?(A) 2 (B) 5 (C) 10 (D) 15 (E) 20中文:一张展览票全价为20美元。

Susan用优惠券买4张票打七五折。

Pam用优惠券买5张票打七折。

Pam比Susan多花了多少美元?2、An aquarium has a rectangular base that measures 100cm by 40cm and has a height of 50cm. It is filled with water to a height of 40cm. A brick with a rectangular base that measures 40cm by 20cm and a height of 10cm is placed in the aquarium. By how many centimeters does that water rise?(A) 0.5 (B) 1 (C) 1.5 (D) 2 (E)2.5中文:一个养鱼缸有100cm×40cm的底,高为50cm。

它装满水到40cm的高度。

把一个底为40cm×20cm,高为10cm的砖块放在这个养鱼缸里。

鱼缸里的水上升了多少厘米?3、The larger of two consecutive odd integers is three times the smaller. What is their sum?(A) 4 (B) 8 (C) 12 (D) 16 (E) 20中文:2个连续的奇整数中较大的数是较小的数的3倍。

AMC10美国数学竞赛讲义全

AMC10美国数学竞赛讲义全

AMC10美国数学竞赛讲义全AMC 中的数论问题1:Remember the prime between 1 to 100:2 3 5 7 11 13 17 19 23 29 31 37 41 43 47 53 59 61 67 71 73 79 83 89 91 2:Perfect number:Let is the prime number.if21p - is also the prime number. then1(21)2p p --is the perfect number. For example:6,28,496.3: Let ,0n abc a =≠ is three digital integer .if 333n a b c =++Then the number n is called Daffodils number . There are only four numbers: 153 370 371 407 Let ,0n abcd a =≠ is four digital integer .if 4444d n a b c +=++Then the number n is called Roses number . There are only three numbers: 1634 8208 94744:The Fundamental Theorem of ArithmeticEvery natural number can be written as a product of primes uniquely up to order.5:Suppose that a and b are integers with b =0. Then there exists unique integers q a d such that 0 ≤ < |b| a d a = bq + .6:(1)G eatest Commo D v so : Let gcd (a, b) = max {d ∈ Z: d | a a d d | b}. For any integers a and b, we havegcd(a, b) = gcd(b, a) = gcd(±a, ±b) = gcd(a, b ? a) = gcd(a, b + a). For example: gcd(150, 60) = gcd(60, 30) = gcd(30, 0) = 30 (2)Least commo mult le:Let lcm(a,b)=m {d∈Z: a | d a d b | d }. (3)We have that: ab= gcd(a, b) lcm(a,b) 7:Congruence modulo nIf ,0a b mq m -=≠,then we call a congruence b modulo m and we rewritemod a b m ≡.(1)Assume a b c d m Z 0 m 0 .If a b mod m c d mod m then we havemod a c b d m ±≡± , mod ac bd m ≡ , mod k k a b m ≡(2) The equat o ax ≡ b (mod m) has a solut o f a d o ly f gcd(a, m) d v des b.8:How to find the unit digit of some special integers (1)How many zero at the end of !nFor example, when 100n =, Let N be the number zero at the end of 100!then10010010020424525125N=++=+=(2) ,,a n Z ∈Find the unit digit n a . For example, when 100,3n a ==9:Palindrome, such as 83438, is a number that remains the same when its digits are reversed.There are some number not only palindrome but 2 2 ,222 , (1)Some special palindrome n that 2n is also palindrome. For example :222221111121111123211111123432111111111112345678987654321=====(2)How to create a palindrome? Almost integer plus the number of its reverseddigits and repeat it again and again. Then we get a palindrome. For example: 87781651655617267266271353135335314884+=+=+=+=But whether any integer has this Property has yet to prove(3) The palindrome equation means that equation from left to right and right to left it all set up. For example :1242242112231132211121241388888831421211====Let ab and cde are two digital and three digital integers. If the digits satisfy the,,9a c b e d c e d ?=?=+≤, then ab cde edc ba ?=?.10: Features of an integer divisible by some prime number If n is even ,then 2|n⼀个整数n 的所有位数上的数字之和是3(或者9)的倍数,则n 被3(或者9)整除⼀个整数n 的尾数是零,则n 被5整除⼀个整数n 的后三位与截取后三位的数值的差被7、11、13整除,则n 被7、11、13整除⼀个整数n 的最后两位数被4整除,则n 被4整除⼀个整数n 的最后三位数被8整除,则n 被8整除⼀个整数n 的奇数位之和与偶数位之和的差被11整除,则n 被11整除 11. The number Theoretic functions If 312123t rr rrt n p p p p =(1) {}12()#0:|(1)(1)(1)t n a a n r r r χ=>=+++(2) 12222111222|()(1)(1)(1)t r r r t t t a nn a p pp p p p p p p δ==+++++++++∑(3) {}11221111122()#:,gcd(,)1()()()t t r r rr rr t t n a N a n a n p p p p p p φ---=∈≤==---For example: 2(12)(23)(21)(11)6χχ=?=++= 22(12)(23)(122)(13)28δδ=?=+++= 22(12)(23)(22)(31)4φφ=?=--= Exercise1. The sums of three whole numbers taken in pairs are 12, 17, and 19. What is the middle number?(A) 4 (B) 5 (C) 6 (D) 7(E) 83. For the positive integer n, let denote the sum of all the positive divisors of n with the exception of n itself. For example,<4>=1+2=3 and <12>=1+2+3+4+6=16. What is <<<6>>>?(A) 6 (B) 12 (C) 24 (D) 32 (E) 36 8. What is the sum of all integer solutions to 21<(x-2)<25? (A) 10 (B) 12(C) 15(D) 19(E) 5(A) 6 (B) 7 (C) 8 (D) 9 (E) 10(A) 1(B) 2(C) 3(D) 4(E) 515.The figures 123,,F F F and 4F shown are the first in a sequence of figures. For3n ≥, n F is constructed from -1n F by surrounding it with a square and placing one more diamond on each side of the new square than -1n F had on each side of its outside square. For example, figure 3F has 13 diamonds. How many diamonds are there in figure 20F ?18. Positive integers a, b, and c are randomly and independently selected with replacement from the set {1, 2, 3,…, 2010}. What is the probability that abc ab a ++ is divisible by 3? (A)13(B)2981(C)3181(D)1127(E)132724. Let ,a b and c be positive integers with >>a b c such that 222-b -c +=2011a ab and222+3b +3c -3-2-2=-1997a ab ac bc . What is a ?(A) 249 (B) 250 (C) 251 (D) 252 (E)2535. In multiplying two positive integers a and b, Ron reversed the digits of the two-digit number a. His erroneous product was 161. What is the correct value of the product of a and b?(A) 116 (B) 161 (C) 204 (D) 214 (E) 224 23. What is the hundreds digit of 20112011?(A) 1 (B) 4 (C) 5 (D) 6 (E) 99. A palindrome, such as 83438, is a number that remains the same when its digits are reversed. The numbers x and x+32 are three-digit and four-digit palindromes, respectively. What is the sum of the digits of x?(A) 20 (B) 21 (C) 22 (D) 23 (E) 2421. The polynomial 322010x ax bx -+- has three positive integer zeros. What is the smallest possible value of a?(A) 78 (B) 88 (C) 98 (D) 108 (E) 11824. The number obtained from the last two nonzero digits of 90! Is equal to n. What is n?(A) 12 (B) 32 (C) 48 (D) 52 (E) 6825. Jim starts with a positive integer n and creates a sequence of numbers. Each successive number is obtained by subtracting the largest possible integer square less than or equal to the current number until zero is reached. For example, if Jim starts with n=55, then his sequence contain 5 numbers:55 55-72= 6 6-22= 2 2-12= 1 1-12= 0Let N be the smallest umbe fo wh ch J m’s seque ce has umbe s. What is the units digit of N?(A) 1 (B) 3 (C) 5 (D) 7 (E) 9 21.What is the remainder when 01220093+3+3++3is divided by 8?(A) 0 (B) 1 (C) 2 (D) 4 (E) 65.What is the sum of the digits of the square of 111,111,111? (A) 18 (B) 27(C) 45(D) 63(E) 81ABC 100643a S 2(A) 6 (B) 7(C) 8(D) 9(E) 1024. Let 2200820082k =+. What is the units digit of 222k +? (A) 0 (B) 1(C) 4(D) 6(E) 8AMC about algebraic problems⼀、Linear relations(1) Slope y-intercept form: y kx b =+ (k is the slope, b is the y-intercept) (2)Standard form: 0Ax By C ++= (3)Slope and one point 0000(,),()()P x y k slope y y k x x -=- (4) Two points 1122(,),(,)P x y P x y12121212y y y y y y x x x x x x ---==--- (5)x,y-intercept form: (,0),(0,),(0,0)1x yP a Q b a b a b≠≠+= ⼆、the relations of the two lines 111222:0,:0l A x B y C l A x B y C ++=++=(1) 1l ∥2l 122112210,0A B A B C B C B ?-=-≠ (1) 1l ⊥2l 12120A A B B ?-=三、Special multiplication rules:222223322332212211222112222()()()2()()()()()()(2)()((1)(1))(1)()n n n n n n n n n n n n n n a b a b a b a b a ab b a b a b a ab b a b a b a ab b a b a b a a b ab b n a b a b a a b ab b n is odd n a b c ab bc ac a b -----------=-+±=±+-=-+++=+-+-=-++++≥+=+-++-+->++=++?-22()()0b c c a a b c+-+-=?==四、quadratic equations and PolynomialThe quadratic equations 2(0)y ax bx c a =++≠ has two roots 12,x x then we has1212b c x x x x a a+=-=More generally, if the polynomial 121210nn n n n x a x a x a x a ---+++++= has nroots 123,,,,n x x x x ,then we have:1231122312123(1)n n n n n nx x x x a x x x x x x a x x x x a -++++=-++==-开⽅的开⽅、估计开⽅数的⼤⼩绝对值⽅程Arithmetic Sequence123(1)(2)(3)()n m a a n d a n d a n d a n m d =+-=+-=+-==+-121321()()()()2222n n n m n m n n a a n a a n a a n a a s ---+++++=====1(1)2n n n ds na -=+If n=2k, then we have 1()n k k s k a a +=+ If n=2k+1, then we have 1n k s na += Geometric sequence123123n n n n m n m a a q a q a q a q ----=====1(1)1,1n n a q q s q-≠=-Some special sequence , , 2, 3, 5, ,… 9,99,999,9999,… 1,11,111,1111,… Exercise4 .When Ringo places his marbles into bags with 6 marbles per bag, he has 4 marbles left over. When Paul does the same with his marbles, he has 3 marbles left over. Ringo and Paul pool their marbles and place them into as many bags as possible, with 6 marbles per bag. How many marbles will be left over?7 For a science project, Sammy observed a chipmunk and a squirrel stashing acorns in holes. The chipmunk hid 3 acorns in each of the holes it dug. The squirrel hid 4 acorns in each of the holes it dug. They each hid the same number of acorns, although the squirrel needed 4 fewer holes. How many acorns did the chipmunk hide?21. Four distinct points are arranged on a plane so that the segments connecting them have lengths ,,,,, and . What is the ratio of to ?13. An iterative average of the numbers 1, 2, 3, 4, and 5 is computed the following way. Arrange the five numbers in some order. Find the mean of the first two numbers, and then find the mean of that with the third number, then the mean of that with the fourth number, and finally the mean of that with the fifth number. What is the difference between the largest and smallest possible values that can be obtained using this procedure?16. Three runners start running simultaneously from the same point on a 500-meter circular track. They each run clockwise around the course maintaining constant speeds of 4.4, 4.8, and 5.0 meters per second. The runners stop once they are all together again somewhere on the circular course. How many seconds do the runners run?24. Let ,a b and c be positive integers with >>a b c such that 222-b -c +=2011a ab and222+3b +3c -3-2-2=-1997a ab ac bc . What is a ?(A) 249 (B) 250(C) 251(D) 252(E) 2531. What is246135135246++++-++++? (A) -1 (B) 536(C) 712(D)14760(E)43310. Consider the set of numbers {1, 10, 102, 103(010)}. The ratio of the largest element of the set to the sum of the other ten elements of the set is closest to which integer?(A) 1 (B) 9 (C) 10 (D) 11 (E) 101=(A) -64 (B) -24 (C) -9 (D) 24 (E) 5764. Let X and Y be the following sums of arithmetic sequences: X= 10 + 12 + + …+ 00. Y= 2 + + + …+ 02. What is the value of Y X -?(A) 92 (B) 98 (C) 100 (D) 102 (E) 1127. Which of the following equations does NOT have a solution?(A) 2(7)0x +=(B) -350x += 20=80= (E) -340x -=(A)(B)(C)2(D) (E) 613. What is the sum of all the solutions of 2602x x x =--?(A) 32 (B) 60 (C) 92 (D) 120 (E) 12414. The average of the numbers 1, 2, 3… 9 , 99, and x is 100x. What is x?(A)49101(B)50101(C)12 (D)51101(E)509911. The length of the interval of solutions of the inequality 23a x b ≤+≤ is 10.What is b-a?(A) 6 (B) 10 (C) 15 (D) 20 (E) 3013. Angelina drove at an average rate of 80 kph and then stopped 20 minutes for gas. After the stop, she drove at an average rate of 100 kph. Altogether she drove 250 km in a total trip time of 3 hours including the stop. Which equation could be used to solve for the time t in hours that she drove before her stop? (A) 880100()2503t t +-=(B) 80250t = (C) 100250t =(D) 90250t =(E) 880()1002503t t -+=21. The polynomial 32-2010x ax bx +- has three positive integer zeros. What is the smallest possible value of a?(A) 78 (B) 88 (C) 98 (D) 108 (E) 118 15.When a bucket is two-thirds full of water, the bucket and water weigh kilograms. When the bucket is one-half full of water the total weight is kilograms. In terms of and , what is the total weight in kilograms when the bucket is full of water?13.Suppose thatand. Which of the following is equal tofor every pair of integers16.Let ,,, and be real numbers with,, and. What is the sum of all possible values of5. Which of the following is equal to the product?81216442008............481242004n n +? (A) 251(B) 502(C) 1004(D) 2008 (E) 40167. The fraction 20082200622007220052(3)(3)(3)(3)-- simplifies to which of the following? (A) 1 (B) 9/4 (C) 3 (D) 9/2 (E) 913. Doug can paint a room in 5 hours. Dave can paint the same room in 7 hours. Doug and Dave paint the room together and take a one-hour break for lunch. Let t be the total time, in hours, required for them to complete the job working together, including lunch. Which of the following equations is satisfied by t ?(A) 11()(1)157t ++=(B) 11()1157t ++= (C) 11()157t +=(D) 11()(1)157t +-=(E) (57)1t +=15. Yesterday Han drove 1 hour longer than Ian at an average speed 5 miles per hour faster than Ian. Jan drove 2 hours longer than Ian at an average speed 10 miles per hour faster than Ian. Han drove 70 miles more than Ian. How many more miles did Jan drive than Ian?(A) 120 (B) 130 (C) 140 (D) 150 (E) 160AMC 中的⼏何问题⼀、三⾓形有关知识点1.三⾓形的简单性质与⼏个⾯积公式①三⾓形任何两边之和⼤于第三边;②三⾓形任何两边之差⼩于第三边;③三⾓形三个内⾓的和等于 0°;④三⾓形三个外⾓的和等于3 0°;⑤三⾓形⼀个外⾓等于和它不相邻的两个内⾓的和;⑥三⾓形⼀个外⾓⼤于任何⼀个和它不相邻的内⾓。

【国家自然科学基金】_过载保护_基金支持热词逐年推荐_【万方软件创新助手】_20140803

【国家自然科学基金】_过载保护_基金支持热词逐年推荐_【万方软件创新助手】_20140803

2013年 序号 1 2 3 4 5 6 7 8 9 10 11 12 13 14 15 16 17 18 19 20 21 22 23 24
科研热词 风力发电 过载保护 轻型高压直流输电 负荷减载 负刚度 线路过负荷 紧急控制 电磁环网 电池储能系统 电压源换流器 生机界面 潮流跟踪 暂态电压安全 接受腔 循环功率 并网逆变器 平滑时间常数 多端网络 发电机减出力 压力可变 功率限幅 功率转移比 低压减载 优化控制
推荐指数 1 1 1 1 1 1 1 1 1 1 1 1 1 1 1 1 1 1 1 1 1 1 1 1
2010年 序号 1 2 3 4
科研热词 糖尿病 生物地质环境 地球化学元素 发病率
ቤተ መጻሕፍቲ ባይዱ推荐指数 1 1 1 1
2011年 序号 1 2 3 4 5 6 7 8 9 10 11 12 13 14 15 16 17 18 19 20 21 22 23 24 25 26 27 28 29 30 31 32 33 34 35 36 37 38 39 40
2012年 序号 1 2 3 4 5 6 7 8 9 10 11 12 13 14 15 16 17 18 19 20 21 22 23 24 25 26 27 28 29 30 31 32 33 34 35 36 37 38 39 40 41 42 43 44 45 46 47 48 49 50 51 52
2011年 科研热词 频率稳定 非线性接触 非同期并列 隔离 限流可调 重叠分解 部分元等效电路 过载控制 过载保护 过载 输电塔 负荷/频率控制(lfc) 耦合器 线性二次最优控制 直流稳压电源 疲劳 电路建模 电磁式 牺牲层技术 极限承载力 无接触 探针 性能测试 寄生参数 安全销 外旁通柜 备自投 增量加载 压力传感器 单片机 协调控制 区域备自投 准入控制 交直流系统 串供 三维微力 session粒度 mems传感器 flash crowd ansoft 推荐指数 1 1 1 1 1 1 1 1 1 1 1 1 1 1 1 1 1 1 1 1 1 1 1 1 1 1 1 1 1 1 1 1 1 1 1 1 1 1 1 1

实用文档之AMC10美国数学竞赛A卷附中文翻译和答案

实用文档之AMC10美国数学竞赛A卷附中文翻译和答案

实用文档之"2011AMC10美国数学竞赛A卷"1. A cell phone plan costs $20 each month, plus 5¢per text message sent, plus 10¢ for each minute used over 30 hours. In January Michelle sent 100 text messages and talked for 30.5 hours. How much did she have to pay?(A) $24.00 (B) $24.50 (C) $25.50 (D) $28.00 (E) $30.002. A small bottle of shampoo can hold 35 milliliters of shampoo, Whereas a large bottle can hold 500 milliliters of shampoo. Jasmine wants to buy the minimum number of small bottles necessary to completely fill a large bottle. How many bottles must she buy?(A) 11 (B) 12 (C) 13 (D) 14 (E) 153. Suppose [a b] denotes the average of a and b, and {a b c} denotes the average of a, b, and c. What is {{1 1 0} [0 1] 0}?(A) 29(B)518(C)13(D) 718(E) 234. Let X and Y be the following sums of arithmetic sequences:X= 10 + 12 + 14 + …+ 100.Y= 12 + 14 + 16 + …+ 102.What is the value of Y X -?(A) 92(B) 98 (C) 100 (D) 102 (E) 1125. At an elementary school, the students in third grade, fourth grade, and fifth grade run an average of 12, 15, and 10 minutes per day, respectively. There are twice as many third graders as fourth graders, and twice as many fourth graders as fifth graders. What is the average number of minutes run per day by these students?(A) 12 (B) 373 (C) 887 (D) 13 (E) 146. Set A has 20 elements, and set B has 15 elements. What is the smallest possible number of elements in A ∪B, the union ofA and B?(A) 5(B) 15 (C) 20 (D) 35 (E) 3007. Which of the following equations does NOT have a solution?(A) 2(7)0x += (B) -350x += (C) 20=(D) 80x-==(E) -3408. Last summer 30% of the birds living on Town Lake were geese, 25% were swans, 10% were herons, and 35% were ducks. What percent of the birds that were not swans were geese?(A) 20 (B) 30 (C) 40 (D) 50 (E) 609. A rectangular region is bounded by the graphs of the equations y=a, y=-b, x=-c, and x=d, where a, b, c, and d are all positive numbers. Which of the following represents the area of this region?(A) ac + ad + bc + bd (B) ac – ad + bc – bd (C) ac + ad – bc – bd(D) –ac –ad + bc + bd (E) ac – ad – bc + bd10. A majority of the 20 students in Ms. Deameanor’s class bought pencils at the school bookstore. Each of these students bought the same number of pencils, and this number was greater than 1. The cost of a pencil in cents was greater than thenumber of pencils each student bought, and the total cost of all the pencils was $17.71. What was the cost of a pencil in cents?(A) 7(B) 11 (C) 17 (D) 23 (E) 7711. Square EFGH has one vertex on each side of square ABCD. Point E is on AB with AE=7·EB. What is the ratio of the area of EFGH to the area of ABCD?(A)4964 (B) 2532 (C) 78 (D) 8 (E)412. The players on a basketball team made some three-point shots, some two-point shots, some one-point free throws. They scored as many points with two-point shots as with three-point shots. Their number of successful free throws was one more than their number of successful two-point shots. The team’s total score was 61 points. How many free throws did they make?(A) 13(B) 14 (C) 15 (D) 16 (E) 1713. How many even integers are there between 200 and 700 whose digits are all different and come from the set {1, 2, 5, 7, 8, 9}?(A) 12 (B)20 (C)72 (D) 120 (E) 20014. A pair of standard 6-sided fair dice is rolled once. The sum of the numbers rolled determines the diameter of a circle. What is the probability that the numerical value of the area of the circle is less than the numerical value of the circle’s circumference?(A) 136(B)112(C)16(D) 14(E) 51815. Roy bought a new battery-gasoline hybrid car. On a trip the car ran exclusively on its battery for the first 40 miles, then ran exclusively on gasoline for the rest of the trip, using gasoline at a rate of 0.02 gallons per mile. On the whole trip he averaged 55 miles per gallon. How long was the trip in miles?(A) 140 (B) 240 (C) 440 (D) 640 (E) 84016. Which of the following in equal to(A)(B) (C) 2 (D) (E)617. In the eight-term sequence A, B, C, D, E, F, G , H, the value of C is 5 and the sum of any three consecutive terms is 30. What is A + H?(A) 17(B) 18 (C) 25 (D) 26 (E) 4318. Circles A, B, and C each have radius 1. Circles A and B share one point of tangency. Circle C has a point of tangency with the midpoint of AB. What is the area inside Circle C but outside Circle A and Circle B? (A) 32π- (B) 2π (C) 2 (D) 34π (E) 12π+19. In 1991 the population of a town was a perfect square. Ten years later, after an increase of 150 people, the population was 9 more than a perfect square. Now, in 2011, with an increase of another 150 people, the population is once again a perfectsquare. Which of the following is closest to the percent growth of the town’s population during this twenty-year period? (A) 42 (B) 47 (C) 52 (D) 57 (E) 6220. Two points on the circumference of a circle of radius r are selected independently and at random. From each point a chord of length r is drawn in a clockwise direction. What is the probability that the two chords intersect?(A) 16(B) 15(C) 14(D) 13(E) 1221. Two counterfeit coins of equal weight are mixed with 8 identical genuine coins. The weight of each of the counterfeit coins is different from the weight of each of the genuine coins.A pair of coins is selected at random without replacement from the 10 coins. A second pair is selected at random without replacement from the remaining 8 coins. The combined weight of the first pair is equal to the combined weight of the second pair. What is the probability that all 4 selected coins are genuine?(A) 711(B) 913(C) 1115(D) 1519(E) 151622. Each vertex of convex pentagon ABCDE is to be assigned a color. There are 6 colors to choose from, and the ends of each diagonal must have different colors. How many different colorings are possible?(A) 2500 (B) 2880 (C) 3120 (D) 3250 (E) 375023. Seven students count from 1 to 1000 as follows:·Alice says all the numbers, except she skips the middle number in each consecutive group of three numbers. That is Alice says 1, 3, 4, 6, 7, 9, …, 997, 999, 1000.·Barbara says all of the numbers that Alice doesn’t say, except she also skips the middle number in each consecutive grope of three numbers.·Candice says all of the numbers that neither Alice nor Barbara says, except she also skips the middle number in each consecutive group of three numbers.·Debbie, Eliza, and Fatima say all of the numbers that none of the students with the first names beginning before theirs inthe alphabet say, except each also skips the middle number in each of her consecutive groups of three numbers.·Finally, George says the only number that no one else says. What number does George say?(A) 37(B) 242 (C) 365 (D) 728 (E) 99824. Two distinct regular tetrahedra have all their vertices among the vertices of the same unit cube. What is the volume of the region formed by the intersection of the tetrahedra?(A)112 (B) 12 (C) (D) 16 (E)25. Let R be a square region and 4n an integer. A point X in the interior of R is called n-ray partitional if there are n rays emanating from X that divide R into N triangles of equal area. How many points are 100-ray partitional but not 60-ray partitional?(A) 1500(B) 1560 (C) 2320 (D) 2480 (E)25002011AMC10美国数学竞赛A卷1. 某通讯公司手机每个月基本费为20美元, 每传送一则简讯收 5美分(一美元=100 美分)。

2008 AMC10A中文解析

2008 AMC10A中文解析

AMC10A 2008 精品解析Q1. A bakery owner turns on his doughnut machine at 8:30 AM. At 11:10 AM the machine has completed one third of the day’s job. At what time will the doughnut machine complete the job?A) 1:50 PM B) 3:00 PM C)3:30 PM D)4:30 PM E) 5:50 PM 翻译:面包店老板在早上8:30打开他的甜甜圈机。

上午11:10,机器完成了一天工作的三分之一。

甜甜圈机什么时候能够完成工作?答案:D解析:由题意,从8:30到11:10完成了工作的三分之一,共花费了160分钟,若完成整项工作,需要花费480分钟,在4:30 PM 完成,故选D.Q2. A square is drawn inside a rectangle. The ratio of the width of the rectangle to a side of the square is 2 : 1. The ratio of the rectangle’s length to its width is 2 : 1. What percent of the rectangle’s area is inside the square?A) 12.5 B) 25 C)50 D)75 E)87.5 翻译:在矩形内画一个正方形。

矩形的宽度与正方形的边长之比是2:1。

这个矩形的长宽比是2:1。

问正方形面积是矩形面积的百分之多少?答案:A解析:设正方形的边长为a ,则由题意,矩形的宽为2a ,长为4a ,所以正方形的面积为2a ,矩形的面积为28a ,正方形面积是矩形面积的百分之12.5,故选A.Q3. For the positive integer n , let n denote the sum of all the positive divisors of n with the exception of n itself. For example,4123=+= and 121+2+3+4+6=16=, What is 6?A) 6 B) 12 C)24 D)32 E)36 翻译:对于正整数n ,n 表示除n 本身外所有正因数的和。

2008年 2009 美国AMC10

2008年 2009 美国AMC10

2008年 第9届 美国AMC10(2008年2月 日 时间75分钟)1. 某面包店老板在上午8:30启动甜甜圈制作机,在上午11:10这机器已完成当天三分之一的工作。

请问该机器何时可以完成当天全部的工作?(A) 下午1:50 (B) 下午3:00 (C) 下午3:30 (D) 下午4:30 (E) 下午5:50 。

2. 在长方形内昼一个正方形。

长方形的宽与正方形的边长比为2:1,长方形的长与宽之比为2:1。

请问正方形面积占长方形面积的百分之多少? (A) 12.5 (B) 25 (C) 50 (D) 75 (E) 87.5 。

3. 对于正整数n ,以<n >表示除了n 本身之外其所有正因子的和。

例如:<4>=1+2=3,而<12>=1+2+3+4+6=16。

请问<<<6>>>=? (A) 6 (B) 12 (C) 24 (D) 32 (E) 36 。

4. 若10根香蕉价钱的32与8个橘子的价钱相同,请问多少个橘子的价钱会与5根香蕉价钱的21相同? (A) 2 (B) 25 (C) 3 (D)27 (E) 4 。

5. 请问分式48⨯812⨯…⨯nn 444+⨯…⨯20042008的乘积是多少?(A) 251 (B) 502 (C) 1004 (D) 2008 (E) 4016 。

6. 某选手参加游泳、单车、赛跑三项距离都相等的三项运动竞赛,这名选手游泳的速率为每小时3公里,骑单车的速率为每小时20公里,跑步的速率为每小时10公里。

请问下列何者最接近这位选手整个赛程的平均速率(公里/小时)? (A) 3 (B) 4 (C) 5 (D) 6 (E) 7 。

7. 请问分式22005220072200622008)3()3()3()3(--可化简为何? (A) 1 (B)49 (C) 3 (D)29 (E) 9 。

8. 小华比较某种新计算机在两家商店的价格。

2010_2015年AMC10A和B竞赛真题与答案(英文版)

2010_2015年AMC10A和B竞赛真题与答案(英文版)
2013年AMC 10A竞赛真题及答案(英文版)
201052015年aamc10a和bb竞赛真题及答案英文版你好目前只分享了20102013年amc10a和b竞赛真题及答案英文版20142015年的暂时还没有上传等采纳后再私信我吧
2010-2015年AMC 10A和B竞赛真题及答案(英文版)
你好,目前,只分享了2010-2013年AMC 10A和B竞赛真题及答案(英文版),2014-2015年的暂时还没有上传,等采纳后再私信我吧!需要的话,我可以将Word文档中的原图文件一同发给你,求采纳,毕竟我花了差不多一个下午才整理完,谢谢!(@_@)
2010年AMC 10A竞赛真题及答案(英文版)
2010年AMC 10B竞赛真题及答赛真题及答案(英文版)
2011年AMC 10B竞赛真题及答案(英文版)
Problem 4
2012年AMC 10A竞赛真题及答案(英文版)
2012年AMC 10B竞赛真题及答案(英文版)

2010 AMC 12A Problems

2010 AMC 12A Problems

2010 AMC 12A ProblemsProblem 1What is ?Problem 2A ferry boat shuttles tourists to an island every hour starting at 10 AM until its last trip, which starts at 3 PM. One day the boat captain notes that on the 10 AM trip there were 100 tourists on the ferry boat, and that on each successive trip, the number of tourists was 1 fewer than on the previous trip. How many tourists did the ferry take to the island that day?Problem 3Rectangle , pictured below, shares of its area with square .Square shares of its area with rectangle . What is ?If , then which of the following must be positive?Problem 5Halfway through a 100-shot archery tournament, Chelsea leads by 50 points. For each shot a bullseye scores 10 points, with other possible scores being 8, 4, 2, and 0 points. Chelsea always scores at least 4 points on each shot. If Chelsea's next shots are bullseyes she will be guaranteed victory. What is the minimum value for ?Problem 6A , such as 83438, is a number that remains the same when its digits are reversed. The numbers and are three-digit and four-digitpalindromes, respectively. What is the sum of the digits of ?Problem 7Logan is constructing a scaled model of his town. The city's water tower stands 40 meters high, and the top portion is a sphere that holds 100,000 liters of water. Logan's miniature water tower holds 0.1 liters. How tall, in meters, should Logan make his tower?Triangle has . Let and be on and , respectively,such that . Let be the intersection of segments and, and suppose that is equilateral. What is ?Problem 9A solid cube has side length inches. A -inch by -inch square hole is cut intothe center of each face. The edges of each cut are parallel to the edges of the cube, and each hole goes all the way through the cube. What is the volume, in cubic inches, of the remaining solid?Problem 10The first four terms of an arithmetic sequence are , , , and .What is the term of this sequence?Problem 11The of the equation can be expressed in the form . What is ?In a magical swamp there are two species of talking amphibians: toads, whose statements are always true, and frogs, whose statements are always false. Four amphibians, Brian, Chris, LeRoy, and Mike live together in this swamp, and they make the following statements.Brian: "Mike and I are different species."Chris: "LeRoy is a frog."LeRoy: "Chris is a frog."Mike: "Of the four of us, at least two are toads."How many of these amphibians are frogs?Problem 13For how many integer values of do the graphs of and notintersect?Problem 14Nondegenerate has integer side lengths, is an angle bisector,, and . What is the smallest possible value of the perimeter?A coin is altered so that the probability that it lands on heads is less than and when the coin is flipped four times, the probaiblity of an equal number of headsand tails is . What is the probability that the coin lands on heads?Problem 16Bernardo randomly picks 3 distinct numbers from the setand arranges them in descending order to form a 3-digit number. Silvia randomlypicks 3 distinct numbers from the set and also arranges them in descending order to form a 3-digit number. What is the probability that Bernardo's number is larger than Silvia's number?Problem 17Equiangular hexagon has side lengths and. The area of is of the area of the hexagon.What is the sum of all possible values of ?A 16-step path is to go from to with each step increasing either the-coordinate or the -coordinate by 1. How many such paths stay outside or onthe boundary of the square , at each step?Problem 19Each of 2010 boxes in a line contains a single red marble, and for , the box in the position also contains white marbles. Isabella begins at thefirst box and successively draws a single marble at random from each box, inorder. She stops when she first draws a red marble. Let be the probabilitythat Isabella stops after drawing exactly marbles. What is the smallest value offor which ?Problem 20Arithmetic sequences and have integer terms withand for some . What is the largest possible value of ?Problem 21The graph of lies above the lineexcept at three values of , where the graph and the line intersect. What is the largest of these values?Problem 22What is the minimum value of?Problem 23The number obtained from the last two nonzero digits of is equal to . Whatis ?Problem 24Let . The intersectionof the domain of with the interval is a union of disjoint open intervals. What is ?Problem 25Two quadrilaterals are considered the same if one can be obtained from the other by a rotation and a translation. How many different convex cyclic quadrilaterals are there with integer sides and perimeter equal to 32?。

【电子技术应用】_系列产品_期刊发文热词逐年推荐_20140725

【电子技术应用】_系列产品_期刊发文热词逐年推荐_20140725

推荐指数 3 3 3 2 2 2 2 2 2 2 2 2 1 1 1 1 1 1 1 1 1 1 1 1 1 1 1 1 1 1 1 1 1 1 1 1 1 1 1 1 1 1 1 1 1 1 1 1 1 1 1 1
2009年 序号 1 2 3 4 5 6 7 8 9 10 11 12 13 14 15 16 17 18 19 20 21 22 23 24 25 26 27 28 29 30 31 32 33 34 35 36 37 38 39 40 41 42 43 44 45 46 47 48 49 50 51 52
泰克公司 气体传感器 服务 最佳解决方案 新时代 新产品 断流 数据中心 数字信号 收购 收发器 支持 拓展 抗干扰性 抄表系统 技术平台 性能 性价比 德州仪器集团 德州仪器 微软公司 微控制器 微处理器 开发工具 延时 应用需求 应用处理器 平台性能 干扰环境 工业控制器 工业 嵌入式系统设计 嵌入式系统 嵌入式开发 嵌入式平台 嵌入式 导航装置 容量 实时监控 实时 多样化 增强型 可编程逻辑器件 发售 卡尔 单电源供电 单片机 半导体 医疗电子 功率运算放大器 创造美 创新功能 创新产品 内核
1 1 1 1 1 1 1 1 1 1 1 1 1 1 1 1 1 1 1 1 1 1 1 1 1 1 1 1 1 1 1 1 1 1 1 1 1 1 1 1 1 1 1 1 1 1 1 1 1 61 62 63 64 65 66 67 68 69 70 71 72 73 74 75 76 77 78 79 80 81 82 83 84 85 86 87 88 89 90 91 92 93 94 95 96 97 98 99 100 101 102 103 104 105 106
信号完整性 保险丝 便携式设备 便携式电子设备 便携式 供应商 低成本 产品设计 产品模型 产品开发周期 产品尺寸 交通信息频道 串行信号 中档 中国市场 业界 一站式 windows ttbdm ti stratix spi spansion se公司 server semiconductor pic单片机 pc oem mirrorbit mcu mb/s maxim i/o hd freescale embedded dsp coldfire can2.0b bdm b/s avr asic arm9 altera公司 9s12单片机 32位mcu

2008 AMC 12B Problems(答案)

2008 AMC 12B Problems(答案)

2008 AMC 12B ProblemsProblem 1A basketball player made baskets during a game. Each basket was worth either or points. How many different numbers could represent the total points scored by the player?SolutionIf the basketball player makes three-point shots and two-point shots, he scorespoints. Clearly every value of yields a different number of total points. Since he can make any number ofthree-point shots between and inclusive, the number of different point totals is .Problem 2A block of calendar dates is shown. The order of the numbers in the second row is to be reversed. Then the order of the numbers in the fourth row is to be reversed. Finally, the numbers on each diagonal are to be added. What will be the positive difference between the two diagonal sums?SolutionAfter reversing the numbers on the second and fourth rows, the block will look like this:页脚内容1The difference between the two diagonal sums is:.Problem 3A semipro baseball league has teams with players each. League rules state that a player must be paid at leastdollars, and that the total of all players' salaries for each team cannot exceed dollars. What is the maximum possiblle salary, in dollars, for a single player?SolutionWe want to find the maximum any player could make, so assume that everyone else makes the minimum possible and that the combined salaries total the maximum ofThe maximum any player could make is dollars (answer choice C)Problem 4On circle , points and are on the same side of diameter , , and . What is the ratio of the area of the smaller sector to the area of the circle?页脚内容2Solution.Since a circle has , the desired ratio is .Problem 5A class collects dollars to buy flowers for a classmate who is in the hospital. Roses cost dollars each, and carnations cost dollars each. No other flowers are to be used. How many different bouquets could be purchased for exactly dollars?SolutionThe class could send just carnations (25 of them). They could also send 22 carnations and 2 roses, 19 carnations and 4 roses, and so on, down to 1 carnation and 16 roses. There are 9 total possibilities (from 0 to 16 roses, incrementing by 2 at each step), which is answer choice C.Problem 6页脚内容3Postman Pete has a pedometer to count his steps. The pedometer records up tosteps, then flips over toon the next step. Pete plans to determine his mileage for a year. On January Pete sets the pedometer to . During the year, the pedometer flips from to forty-four times. On December the pedometer reads . Pete takes steps per mile. Which of the following is closest to the number of miles Pete walked during the year?SolutionEvery time the pedometer flips, Pete has walked steps. Therefore, Pete has walked a total ofsteps, which is miles, which is closest to answer choice A.Problem 7For real numbers and , define . What is ?SolutionProblem 8页脚内容4Pointsand lie on . The length of is times the length of , and the length of is times thelength of . The length of is what fraction of the length of ?SolutionSince and , .Since and , .Thus, .Problem 9Points and are on a circle of radius and . Point is the midpoint of the minor arc . What is the length of the line segment ?SolutionTrig Solution:Let be the angle that subtends the arc AB. By the law of cosines,页脚内容5The half-angle formula says that, which is answer choice A.Other SolutionDefine D as the midpoint of AB, and R the center of the circle. R, C, and D are collinear, and since D is the midpoint of AB, , and so . Since , , and soProblem 10Bricklayer Brenda would take hours to build a chimney alone, and bricklayer Brandon would take hours to build it alone. When they work together they talk a lot, and their combined output is decreased by bricks per hour. Working together, they build the chimney in hours. How many bricks are in the chimney?SolutionLet be the number of bricks in the house.Without talking, Brenda and Brandon lay and bricks per hour respectively, so together they layper hour together.Since they finish the chimney in hours, . Thus, .页脚内容6Problem 11A cone-shaped mountain has its base on the ocean floor and has a height of 8000 feet. The top of the volume of the mountain is above water. What is the depth of the ocean at the base of the mountain in feet?SolutionIn a cone, radius and height each vary inversely with increasing height (i.e. the radius of the cone formed by cutting off the mountain at feet is half that of the original mountain). Therefore, volume varies as the inverse cube of increasing height (expressed as a percentage of the total height of cone):Plugging in our given condition,, answer choice A.Problem 12For each positive integer , the mean of the first terms of a sequence is . What is the th term of the sequence?SolutionLetting be the nth partial sum of the sequence:页脚内容7The only possible sequence with this result is the sequence of odd integers.Problem 13Vertex of equilateral is in the interior of unit square . Let be the region consisting of all points inside and outside whose distance from is between and . What is the area of ?Problem 14A circle has a radius of and a circumference of . What is ?SolutionLet be the circumference of the circle, and let be the radius of the circle.Using log properties, and .Since , .Problem 15 (no solution)页脚内容8On each side of a unit square, an equilateral triangle of side length 1 is constructed. On each new side of each equilateral triangle, another equilateral triangle of side length 1 is constructed. The interiors of the square and the12 triangles have no points in common. Let be the region formed by the union of the square and all the triangles, and be the smallest convex polygon that contains . What is the area of the region that is inside but outside ?Baidu查的答案答案是A,其实画个图就清楚了,边长为1的正方形(unit square) 连同周边12个正三角形组成一个新的边长为2的正方形,要使包在正方形外面的八边形面积最小,只有A是正确的,BCEDE的话面积都比1/4的时候大。

2010 AMC10A试题(含答案)

2010 AMC10A试题(含答案)

2010 AMC10A ProblemsQ1. Mary’s top book shelf holds five books with the follo wing widths, in centimeters:6,12, 1, 2.5, and 10. What is the average book width, in centimeters?A) 1 B) 2 C) 3 D) 4 E)5Q2. Four identical squares and one rectangle are placed together to form one large square as shown. The length of the rectangle is how many times as large as its width?A)54B)43C)32D) 2 E)3Q3.Tyrone had 97 marbles and Eric had 11 marbles. Tyrone then gave some of his marbles to Eric so that Tyrone ended with twice as many marbles as Eric. How many marbles did Tyrone give to Eric?A)3 B)13 C)18 D) 25 E)29 Q4.A book that is to be recorded onto compact discs takes 412 minutes to read aloud. Each disc can hold up to 56 minutes of reading. Assume that the smallest possible number of discs is used and that each disc contains the same length of reading. How many minutes of reading will each disc contain?A)50.2 B)51.5 C)52.4 D) 53.8 E)55.2Q5.The area of a circle whose circumference is 24π is kπ. What is the value of k?A)6 B)12 C)24 D) 36 E)144Q6. For positive numbers x and y the operation ♠(x, y) is defined as1(,)x y xy♠=-What is ♠(2, ♠(2, 2))?A)23B)1 C)43D)53E)2Q7.Crystal has a running course marked out for her daily run. She starts this run by heading due north for one mile. She then runs northeast for one mile, then southeast for one mile. The last portion of her run takes her on a straight line back to where she started. How far, in miles is this last portion of her run?A)1 B) D)2 E)Q8.Tony works 2 hours a day and is paid $0.50 per hour for each full year of his age. During a six month period Tony worked 50 days and earned $630. How old was Tony at the end of the six month period?A)9 B)11 C)12 D) 13 E)14Q9.A palindrome, such as 83438, is a number that remains the same when its digits are reversed. The numbers x and x + 32 are three -digit and four -digit palindromes, respectively. What is the sum of the digits of x ?A)20 B)21 C)22 D) 23 E)24Q11.The length of the interval of solutions of the inequality 23a x b ≤+≤is 10.What is b a -?A)6 B)10 C)15 D) 20 E)30Q12.Logan is constructing a scaled model of his town. The city’s water tower stands 40 meters high, and the top portion is a sphere that holds 100,000 liters of water. Logan’s miniature water tower holds 0.1 liters. How tall, in meters, should Logan make his tower?A)0.04 B)0.4π C)0.4 D) 4πE)4Q13.Angelina drove at an average rate of 80 kmh and then stopped 20 minutes for gas. After the stop, she drove at an average rate of 100 kmh. Altogether she drove 250 km in a total trip time of 3 hours including the stop. Which equation could be used to solve for the time t in hours that she drove before her stop? A)880100()2503t t +-= B)80250t = C)100250t = D)90250t = E)880()1002503t t -+=Q14.Triangle ABC has 2AB AC =⋅. Let D and E be on AB and BC , respectively, such that ∠BAE = ∠ACD . Let F be the intersection of segments AE and CD , and suppose that △CFE is equilateral. What is ∠ACB ?A)60° B)75° C)90° D) 105° E)120°Q15.In a magical swamp there are two species of talking amphibians: toads, whose statements are always true, and frogs, whose statements are always false. Four amphibians, Brian, Chris, LeRoy, and Mike live together in this swamp, and they make the following statements.Brian: “Mike and I are different species.”Chris: “LeRoy is a frog.”LeRoy: “Chris is a frog.”Mike: “Of the four of us, at least two are toads.”How many of these amphibians are frogs?A)0 B)1 C)2 D) 3 E)4Q16.Nondegenerate △ABC has integer side lengths, BD is an angle bisector, AD = 3, and DC = 8. What is the smallest possible value of the perimeter?A)30 B)33 C)35 D) 36 E)37 Q17.A solid cube has side length 3 inches. A 2-inch by 2-inch square hole is cut into the center of each face. The edges of each cut are parallel to the edges of the cube, and each hole goes all the way through the cube. What is the volume, in cubic inches, of the remaining solid?A)7 B)8 C)10 D) 12 E)15Q18.Bernardo randomly picks 3 distinct numbers from the set {1, 2, 3, 4, 5, 6, 7, 8, 9} and arranges them in descending order to form a 3-digit number. Silvia randomly picks 3 distinct numbers from the set {1, 2, 3, 4, 5, 6, 7, 8} and also arranges them in descending order to form a 3-digit number. What is the probability that Bernardo’s number is larger than Silvia’s number?A)4772B)3756C)23D)4972E)3956Q19.Equiangular hexagon ABCDEF has side lengths AB = CD = EF = 1 and BC = DE = F A = r. The area of △ACE is 70% of the area of the hexagon. What is the sum of all possible values of r?A)3B)103C)4 D)174E) 6Q20.A fly trapped inside a cubical box with side length 1 meter decides to relieve its boredom by visiting each corner of the box. It will begin and end in the same corner and visit each of the other corners exactly once. To get from a corner to any other corner, it will either fly or crawl in a straight line. What is the maximum possible length, in meters, of its path?A)4+B)2+C)2+D)E)Q21.The polynomial 322010-+-has three positive integer roots. What isx ax bxthe smallest possible value of a?A)78 B)88 C)98 D)108 E)118Q22.Eight points are chosen on a circle, and chords are drawn connecting every pair of points. No three chords intersect in a single point inside the circle. How many triangles with all three vertices in the interior of the circle are created?A) 28 B) 56 C) 70 D) 84 E) 140Q23.Each of 2010 boxes in a line contains a single red marble, and for 1 ≤k≤2010, the box in the k th position also contains k white marbles. Isabella begins at the first box and successively draws a single marble at random from each box, in order. She stops when she first draws a red marble. Let P(n) be the probability that Isabella stops after drawing exactly n marbles. What is the smallest value of n for which1()P n<?2010A) 45 B) 63 C) 64 D) 201 E) 1005Q24.The number obtained from the last two nonzero digits of 90! is equal to n. What is n?A) 12 B) 32 C) 48 D) 52 E) 68Q25.Jim starts with a positive integer n and creates a sequence of numbers. Each successive number is obtained by subtracting the largest possible integer square less than or equal to the current number until zero is reached. For example, if Jim starts with n = 55, then his sequence contains 5 numbers:Let N be the smallest number for which Jim’s sequence has 8 numbers. What is the units digit of N?A) 1 B) 3 C) 5 D) 7 E) 9AMC10A 2010中文解析Q1. Mary’s top book shelf holds five books with the follo wing widths, in centimeters:6,12, 1, 2.5, and 10. What is the average book width, in centimeters?A) 1 B) 2 C) 3 D) 4 E)5翻译:Mary最上面的书架上有五本书,宽度分别为: 6厘米、1厘米、2.5厘米和10厘米。

2008年全国英语能力竞赛初二组初赛和决赛试题参考答案

2008年全国英语能力竞赛初二组初赛和决赛试题参考答案

2008年全国中学生英语能力竞赛初赛初二组试题参考答案听力部分(共四大题,计30分)I.Sentences (句子理解)1—5 DCADBII.Responses (问句应答)6—10 BADCBIII. Dialogues (对话理解)A) 11—15 CADBAB) 16—20 ADCDBIV.Passages (短文理解)A) 21—25 BCACDB) 26. Science27. Running28. country(side)29. bike / bicycle30. farm笔试部分(共八大题,计120分)I. Multiple-choice (选择填空)A) 31—35 BCDACB) 36—40 DCACA41—45 BDCDA46—50 CBDABII.Reading comprehension (阅读理解)A)51—55 CBDBAB)A. 56. In a cave.57. He thought the other boys were behind the wall.58. Some old pieces of paper.59. 他(老师)告诉他们,那些纸是地图,而且可能有2000年的历史。

60. b d f a e cB. 61. invitation; Debbie62. big garden63. great / good time64. Green Road65. Baker’sC. 66. animals / creatures67. together68. dies69. shapes70. 它们有着和它们形状相似的(陆地)动物的名字。

III. Cloze (完形填空)A) 71. how far72. find out73. windy74. First75. to76. write back77. their78. who79. later80. balloonsB) 81. walking82. Is83. touch84. to bite85. didn’t lieIV. Sentence pattern transformation (句式转换)86. went into space before people did / went into space earlier than people did87. the second monkey to go into space Enos88. How long did his trip last?89. What did Enos do when the spaceship was opened?90. How happy Enos was to be back on Earth!V. Translation (翻译)91. I will go to the science centre with you if I have time. / If I have time, I will go to the science centre with you.92. Eating a balanced diet / A balanced diet is good for our health.93. It took him an hour to get to the railway station.94. She didn’t leave school until 8 pm yesterday.95. They decided not to go there because of the bad weather.VI. IQ (智力测试)96. stop97. breakfast98. MEET ME AT THE LAB TONIGHT99. It means “can’t understand”. / 弄不明白。

2008 AMC10 A 答案

2008 AMC10 A 答案
This Pamphlet gives at least one solution for each problem on this year’s contest and shows that all problems can be solved without the use of a calculator. When more than one solution is provided, this is done to illustrate a significant contrast in methods, e.g., algebraic vs geometric, computational vs conceptual, elementary vs advanced. These solutions are by no means the only ones possible, nor are they superior to others the reader may devise. We hope that teachers will inform their students about these solutions, both as illustrations of the kinds of ingenuity needed to solve nonroutine problems and as examples of good mathematical exposition. However, the publication, reproduction or communication of the problems or solutions of the AMC 10 during the period when students are eligible to participate seriously jeopardizes the integrity of the results. Dissemination via copier, telephone, e-mail, World Wide Web or media of any type during this period is a violation of the competition rules.

2012-Problems-AMC10B

2012-Problems-AMC10B

INSTRUCTIONS1. DO NOT OPEN THIS BOOKLET UNTIL YOUR PROCTOR TELLS YOU.2. This is a twenty-five question multiple choice test. Each question is followed by answers marked A, B, C, D and E. Only one of these is correct.3. Mark your answer to each problem on the AMC 10 Answer Form with a #2 pencil. Check the blackened circles for accuracy and erase errors and stray marks completely. Only answers properly marked on the answer form will be graded.4. SCORING: You will receive 6 points for each correct answer, 1.5 points for each problem left unanswered, and 0 points for each incorrect answer.5. No aids are permitted other than scratch paper, graph paper, rulers, compass, protractors, and erasers. No calculators are allowed. No problems on the test will require the use of a calculator.6. Figures are not necessarily drawn to scale.7. Before beginning the test, your proctor will ask you to record certain information on the answer form.8. When your proctor gives the signal, begin working on the problems. You will have 75 minutes to complete the test.9. When you finish the exam, sign your name in the space provided on the Answer Form.© 2012 Mathematical Association of AmericaThe Committee on the American Mathematics Competitions (CAMC) reserves the right to re-examine students before deciding whether to grant official status to their scores. The CAMC also reserves the right to disqualify all scores from a school if it is determined that the required security procedures were not followed.Students who score 120 or above or finish in the top 2.5% on this AMC 10 will be invited to take the 30th annual American Invitational Mathematics Examination (AIME) on Thursday, March 15, 2012 or Wednesday, March 28, 2012. More details about the AIME and other information are on the back page of this test booklet.The publication, reproduction or communication of the problems or solutions of the AMC 10 during the period when students are eligible to participate seriously jeopardizes the integrity of the results. Dissemination via copier, telephone, e-mail, World Wide Web or media of any type during this period is a violation of the competition rules. After the contest period, permission to make copies of problems in paper or electronic form including posting on web-pages for educational use is granted without fee provided that copies are not made ordistributed for profit or commercial advantage and that copies bear the copyright notice.**Administration On An Earlier Date Will Disqualify Your School’s Results**1. All information (Rules and Instructions) needed to administer this exam is contained in the TEACHERS’ MANUAL, which is outside of this package. PLEASE READ THE MANUAL BEFORE FEBRUARY 22, 2011. Nothing is needed from inside this package until February 22.2. Your PRINCIPAL or VICE-PRINCIPAL must verify on the AMC 10 CERTIFICATION FORM (found in the Teachers’ Manual) that you fol-lowed all rules associated with the conduct of the exam.3. The Answer Forms must be mailed by trackable mail to the AMC office no later than 24 hours following the exam.4. The publication, reproduction or communication of the problems or solutions of this test during the period when students are eligible to participate seriously jeopardizes the integrity of the results. Dissemination at any time via copier, telephone, e-mail, internet or media of any type is a violation of the competition rules.AMC 10 B DO NOT OPEN UNTIL WEDNEsDAy, FEBRUARy 22, 2012The American Mathematics Competitionsare Sponsored byThe Mathematical Association of America – MAA The Akamai Foundation ContributorsAcademy of Applied Sciences – AAs American Mathematical Association of Two-Year Colleges – AMATyC ...................................................... American Mathematical Society – AMs ........................................................................................................... American Statistical Association – AsA ...................................................................................................... Art of Problem Solving – Awesome Math Casualty Actuarial Society – CAs ................................................................................................................ D.E. Shaw & Co. ................................................................................................................................. IDEA Math Jane Street Math For America Mu Alpha Theta – MAT ....................................................................................................................... National Council of Teachers of Mathematics – NCTM ................................................................................... Pi Mu Epsilon – PME ............................................................................................................................... Society for Industrial and Applied Math - SIAM ............................................................................................ W.H. Freeman and Company ................................................................................................................www. 1.Each third-grade classroom at Pearl Creek Elementary has18students and2pet rabbits.How many more students than rabbits are there in all4of the third-grade classrooms?(A)48(B)56(C)64(D)72(E)802.A circle of radius5is inscribed in a rectangle as shown.The ratio of the lengthof the rectangle to its width is2:1.What is the area of the rectangle?(A)50(B)100(C)125(D)150(E)2003.The point in the xy-plane with coordinates(1000,2012)is reflected across theline y=2000.What are the coordinates of the reflected point?(A)(998,2012)(B)(1000,1988)(C)(1000,2024)(D)(1000,4012)(E)(1012,2012)4.When Ringo places his marbles into bags with6marbles per bag,he has4marbles left over.When Paul does the same with his marbles,he has3marbles left over.Ringo and Paul pool their marbles and place them into as many bags as possible,with6marbles per bag.How many marbles will be left over?(A)1(B)2(C)3(D)4(E)55.Anna enjoys dinner at a restaurant in Washington,D.C.,where the sales tax onmeals is10%.She leaves a15%tip on the price of her meal before the sales tax is added,and the tax is calculated on the pre-tip amount.She spends a total of $27.50for dinner.What is the cost of her dinner without tax or tip?(A)$18(B)$20(C)$21(D)$22(E)$246.In order to estimate the value of x−y where x and y are real numbers withx>y>0,Xiaoli rounded x up by a small amount,rounded y down by the same amount,and then subtracted her rounded values.Which of the following statements is necessarily correct?(A)Her estimate is larger than x−y.(B)Her estimate is smaller than x−y.(C)Her estimate equals x−y.(D)Her estimate equals y−x.(E)Her estimate is0.7.For a science project,Sammy observed a chipmunk and a squirrel stashing acornsin holes.The chipmunk hid3acorns in each of the holes it dug.The squirrel hid4acorns in each of the holes it dug.They each hid the same number of acorns,although the squirrel needed4fewer holes.How many acorns did the chipmunk hide?(A)30(B)36(C)42(D)48(E)548.What is the sum of all integer solutions to1<(x−2)2<25?(A)10(B)12(C)15(D)19(E)259.Two integers have a sum of26.When two more integers are added to thefirsttwo integers the sum is41.Finally when two more integers are added to the sum of the previous four integers the sum is57.What is the minimum number of even integers among the6integers?(A)1(B)2(C)3(D)4(E)510.How many ordered pairs of positive integers(M,N)satisfy the equationM 6=6?(A)6(B)7(C)8(D)9(E)1011.A dessert chef prepares the dessert for every day of a week starting with Sunday.The dessert each day is either cake,pie,ice cream,or pudding.The same dessert may not be served two days in a row.There must be cake on Friday because ofa birthday.How many different dessert menus for the week are possible?(A)729(B)972(C)1024(D)2187(E)230412.Point B is due east of point A .Point C is due north of point B .The distance between points A and C is 10√2meters,and ∠BAC =45◦.Point D is 20meters due north of point C .The distance AD is between which two integers?(A)30and 31(B)31and 32(C)32and 33(D)33and 34(E)34and 3513.It takes Clea 60seconds to walk down an escalator when it is not operating,and only 24seconds to walk down the escalator when it is operating.How many seconds does it take Clea to ride down the operating escalator when she just stands on it?(A)36(B)40(C)42(D)48(E)5214.Two equilateral triangles are contained in a square whose side length is 2√3.The bases of these triangles are the opposite sides of the square,and their intersection is a rhombus.What is the area of the rhombus?(A)32(B)√3(C)2√2−1(D)8√3−12(E)4√3315.In a round-robin tournament with 6teams,each team plays one game against each other team,and each game results in one team winning and one team losing.At the end of the tournament,the teams are ranked by the number of games won.What is the maximum number of teams that could be tied for the most wins at the end of the tournament?(A)2(B)3(C)4(D)5(E)616.Three circles with radius 2are mutually tangent.What is the total area of the circles and the region bounded by them,as shown in the figure?(A)10π+4√3(B)13π−√3(C)12π+√3(D)10π+9(E)13π17.Jesse cuts a circular paper disk of radius12along two radii to form two sectors,the smaller having a central angle of120degrees.He makes two circular cones, using each sector to form the lateral surface of a cone.What is the ratio of the volume of the smaller cone to that of the larger?(A)18(B)14(C)√1010(D)√56(E)√10518.Suppose that one of every500people in a certain population has a particulardisease,which displays no symptoms.A blood test is available for screening for this disease.For a person who has this disease,the test always turns out positive.For a person who does not have the disease,however,there is a2% false positive rate—in other words,for such people,98%of the time the test will turn out negative,but2%of the time the test will turn out positive and will incorrectly indicate that the person has the disease.Let p be the probability that a person who is chosen at random from this population and gets a positive test result actually has the disease.Which of the following is closest to p?(A)198(B)19(C)111(D)4999(E)989919.In rectangle ABCD,AB=6,AD=30,and G is the midpoint of AD.SegmentAB is extended2units beyond B to point E,and F is the intersection of ED and BC.What is the area of BF DG?(A)1332(B)67(C)1352(D)68(E)137220.Bernardo and Silvia play the following game.An integer between0and999,inclusive,is selected and given to Bernardo.Whenever Bernardo receives a number,he doubles it and passes the result to Silvia.Whenever Silvia receivesa number,she adds50to it and passes the result to Bernardo.The winner isthe last person who produces a number less than1000.Let N be the smallest initial number that results in a win for Bernardo.What is the sum of the digits of N?(A)7(B)8(C)9(D)10(E)1121.Four distinct points are arranged in a plane so that the segments connectingthem have lengths a,a,a,a,2a,and b.What is the ratio of b to a?(A)√3(B)2(C)√5(D)3(E)π22.Let(a1,a2,...,a10)be a list of thefirst10positive integers such that for each 2≤i≤10either a i+1or a i−1or both appear somewhere before a i in the list.How many such lists are there?(A)120(B)512(C)1024(D)181,440(E)362,88023.A solid tetrahedron is sliced offa solid wooden unit cube by a plane passingthrough two nonadjacent vertices on one face and one vertex on the opposite face not adjacent to either of thefirst two vertices.The tetrahedron is discarded and the remaining portion of the cube is placed on a table with the cut surface face down.What is the height of this object?(A)√33(B)2√23(C)1(D)2√33(E)√224.Amy,Beth,and Jo listen to four different songs and discuss which ones theylike.No song is liked by all three.Furthermore,for each of the three pairs of the girls,there is at least one song liked by those two girls but disliked by the third.In how many different ways is this possible?(A)108(B)132(C)671(D)846(E)110525.A bug travels from A to B along the segments in the hexagonal lattice picturedbelow.The segments marked with an arrow can be traveled only in the direction of the arrow,and the bug never travels the same segment more than once.How many different paths are there?(A)2112(B)2304(C)2368(D)2384(E)2400WRITE TO US!Correspondence about the problems and solutions for this AMC 10and orders for publications should be addressed to:American Mathematics CompetitionsUniversity of Nebraska, P.O. Box 81606Lincoln, NE 68501-1606Phone 402-472-2257 | Fax 402-472-6087 | amcinfo@ The problems and solutions for this AMC 10 were prepared by the MAA’s Committee on the AMC 10 and AMC 12 under the direction of AMC 10 Subcommittee Chair:Dr. Leroy Wenstrom2012 AIMEThe 30th annual AIME will be held on Thursday, March 15, with the alternate on Wednesday, March 28. It is a 15-question, 3-hour, integer-answer exam. You will be invited to participate only if you score 120 or above or finish in the top 2.5% of the AMC 10, or if you score 100 or above or finish in the top 5% of the AMC 12. T op-scoring students on the AMC 10/12/AIME will be selected to take the 41st Annual USA Mathematical Olympiad (USAMO) on April 24-25, 2012. The best way to prepare for the AIME and USAMO is to study previous exams. Copies may be ordered as indicated below.PUBLICATIONSA complete listing of current publications, with ordering instructions, is at our web site:。

AMC 10 Problems--2004 A

AMC 10 Problems--2004 A

Problem 1You and five friends need to raise dollars in donations for a charity, dividing thefundraising equally. How many dollars will each of you need to raise?Problem 2For any three real numbers , , and , with , the operation is defined by:What is ?Problem 3Alicia earns 20 dollars per hour, of which is deducted to pay local taxes. How many cents per hour of Alicia's wages are used to pay local taxes?Problem 4What is the value of if ?Problem 5A set of three points is randomly chosen from the grid shown. Each three point set has the same probability of being chosen. What is the probability that the points lie on the same straight line?Problem 6Bertha has 6 daughters and no sons. Some of her daughters have 6 daughters, and the rest have none. Bertha has a total of 30 daughters and granddaughters, and no great-granddaughters. How many of Bertha's daughters and grand-daughters have nodaughters?Problem 7A grocer stacks oranges in a pyramid-like stack whose rectangular base is 5 oranges by8 oranges. Each orange above the first level rests in a pocket formed by four oranges below. The stack is completed by a single row of oranges. How many oranges are in the stack?Problem 8A game is played with tokens according to the following rule. In each round, the player with the most tokens gives one token to each of the other players and also places one token in the discard pile. The game ends when some player runs out of tokens. Players , , and start with 15, 14, and 13 tokens, respectively. How many rounds willthere be in the game?Problem 9In the figure, and are right angles. ,and and intersect at . What is the difference between the areas ofand ?Problem 10Coin is flipped three times and coin is flipped four times. What is the probabilitythat the number of heads obtained from flipping the two fair coins is the same?Problem 11A company sells peanut butter in cylindrical jars. Marketing research suggests that usingwider jars will increase sales. If the diameter of the jars is increased by without altering the volume, by what percent must the height be decreased?Problem 12Henry's Hamburger Heaven offers its hamburgers with the following condiments: ketchup, mustard, mayonnaise, tomato, lettuce, pickles, cheese, and onions. A customer can choose one, two, or three meat patties, and any collection of condiments. How many different kinds of hamburgers can be ordered?Problem 13At a party, each man danced with exactly three women and each woman danced with exactly two men. Twelve men attended the party. How many women attended the party?Problem 14The average value of all the pennies, nickels, dimes, and quarters in Paula's purse is 20 cents. If she had one more quarter, the average would be 21 cents. How many dimes does she have in her purse?Problem 15Given that and , what is the largest possible value of ?Problem 16The grid shown contains a collection of squares with sizes from to .How many of these squares contain the black center square?Problem 17Brenda and Sally run in opposite directions on a circular track, starting at diametrically opposite points. They first meet after Brenda has run 100 meters. They next meet after Sally has run 150 meters past their first meeting point. Each girl runs at a constant speed. What is the length of the track in meters?Problem 18A sequence of three real numbers forms an arithmetic progression with a first term of 9. If 2 is added to the second term and 20 is added to the third term, the three resulting numbers form a geometric progression. What is the smallest possible value for the third term of the geometric progression?Problem 19A white cylindrical silo has a diameter of 30 feet and a height of 80 feet. A red stripe with a horizontal width of 3 feet is painted on the silo, as shown, making two complete revolutions around it. What is the area of the stripe in square feet?Problem 20Points and are located on square so that is equilateral. Whatis the ratio of the area of to that of ?Problem 21Two distinct lines pass through the center of three concentric circles of radii 3, 2, and 1.The area of the shaded region in the diagram is of the area of the unshaded region. What is the radian measure of the acute angle formed by the two lines? (Note:radians is degrees.)Problem 22Square has side length . A semicircle with diameter is constructedinside the square, and the tangent to the semicircle from intersects side at .What is the length of ?Problem 23Circles , , and are externally tangent to each other and internally tangent tocircle . Circles and are congruent. Circle has radius and passes throughthe center of . What is the radius of circle ?Problem 24Let , be a sequence with the following properties.(i) , and(ii) for any positive integer.What is the value of ?Problem 25Three mutually tangent spheres of radius 1 rest on a horizontal plane. A sphere of radius 2 rests on them. What is the distance from the plane to the top of the larger sphere?。

2010AMC12B中文版

2010AMC12B中文版

2010AMC12B中文版[ 2010-9-16 11:28:06 | By: zero ]难度逐题增加第一题:小明在一天内(9小时)参加了两个会议,第一个会议历时45分,第二个会议长度是第一个的两倍,求当日他参加会议时间占总时间的百分比第二题:一个L型图,左边长8,顶边长2,右边长2,下边长5,求面积.第三题:某校组织活动,每个学生都需要缴纳一定费用(X元钱)来参加,X为一个整数.现总共收到高一学生们的48元钱,收到高二学生们的64元钱,问X有几个值.第四题:在一个大月中,周一的数量和周三的数量相同,问该月的第一天有几种可能.第五题:a-(b-(c-(d+e)))=a-b-c-d+e,a=1,b=2,c=3,d=4,求e值。

第六题:在年初的调查中,某班全体同学有50%回答“我喜欢数学”,另外50%回答“我不喜欢数学”;在年终调查中该班同学有70%回答“我喜欢数学”,另外30%回答“我不喜欢数学”。

在年初和年终的调查中,改变回答的人占x%,试问X的最大值与最小值之差。

第七题:小明在普通公路上以20公里/小时的速度驾驶,在高速公路上则以30公里/小时的速度驾驶.他总共驾驶了40分钟,行进了16公里,试问他驶过的普通公路的总长度.第八题:某市所有学校都要送出三名选手参加某数学竞赛,在这次比赛中,每位选手的得分都不相同.某校学生A在该比赛中排名在正中间,他同时位列自己学校派出的三名选手中的第一位.其余两名选手B和C分别位列第37名和第64名,试问这所城市中有多少学校.第九题:n能被20整除,其平方的立方根是整数,其立方的平方根也是整数.试求符合要求的最小的n的值.第十题:1,2,3,4,5,6,...,99,x 是一组数,其平均数为100x,求x值.第十一题:试求在所有对称的四位数(例如3443,2112)中,能被7整除的数占所有数的比例.第十二题:解方程:log(2^(1/2),x^(1/2))+log(2,x)+log(4,x^2)+log(8,x^3)+log(16,x^4)=40 注:log(a,b)中a是底,b是数第十三题:在三角形abc中,cos(2a-b)+sin(a+b)=2 c所对的边长为2,求a所对的边长。

相关主题
  1. 1、下载文档前请自行甄别文档内容的完整性,平台不提供额外的编辑、内容补充、找答案等附加服务。
  2. 2、"仅部分预览"的文档,不可在线预览部分如存在完整性等问题,可反馈申请退款(可完整预览的文档不适用该条件!)。
  3. 3、如文档侵犯您的权益,请联系客服反馈,我们会尽快为您处理(人工客服工作时间:9:00-18:30)。
(A) 4 The (B) 8 of (C) 12 consecutive (D) 16 (E) 20 integers is three larger two odd times the smaller. What is their sum? (A) 4 (B) 8 (C) 12 (D) 16 (E) 20
2007 AMC 10 B, Problem #25— 2007 AMC 12 B, Problem #24— 14b “Play with the restriction: a b + 9a is an integer.”
Solution Answer (A): Let u = a/b. Then the problem is equivalent to finding all positive rational numbers u 14 = k, for some integer k. This equation is equivalent to 9u2 − 9uk + 14 = 0, whose such that u + 9 u solutions are √ 9k ± 81k2 − 504 k 1 u= 9k2 − 56. = ± 18 2 6 √ Hence u is rational if and only if 9k2 − 56 is rational, which is true if and only if 9k2 − 56 is a perfect square. Suppose that 9k2 − 56 = s2 for some positive integer s. Then (3k − s)(3k + s) = 56. The only factors of 56 are 1, 2, 4, 7, 8, 14, 28, and 56, so (3k − s, 3k + s) is one of the ordered pairs (1, 56), (2, 28), (4, 14), or (7, 8). The cases (1, 56) and (7, 8) yield no integer solutions. The cases (2, 28) and (4, 14) yield k = 5 and k = 3, respectively. If k = 5, then u = 1/3 or u = 14/3. If k = 3, then u = 2/3 or u = 7/3. Therefore there are four pairs (a, b) that satisfy the given conditions, namely (1, 3), (2, 3), (7, 3), and (14, 3). OR a 14b = k, in two different forms. First, multiply both sides by b and subtract Rewrite the equation + b 9a a to obtain 14b2 = bk − a. 9a Because a, b, and k are integers, 14b2 must be a multiple of a, and because a and b have no common factors greater than 1, it follows that 14 is divisible by a. Next, multiply both sides of the original equation by 9a and subtract 14b to obtain 9a2 = 9ak − 14b. b This shows that 9a2 is a multiple of b, so 9 must be divisible by b. Thus if (a, b) is a solution, then b = 1, 3, or 9, and a = 1, 2, 7, or 14. This gives a total of twelve possible solutions (a, b), each of which can be checked quickly. The only such pairs for which a 14b + b 9a is an integer are when (a, b) is (1, 3), (2, 3), (7, 3), or (14, 3).
AMC 10 Student Practice Questions The larger two consecutive odd integers is three You will find these and additional problemsof for the AMC 10 and AMC 12 on AMC’s web site: /amc, available from the current and previous AMC 10/12 Teacher Manuals, (/amc/e-exams/e6-amc12/archive12.shtml) or times smaller. What is their sum? from our Problems page archives the (/amc/a-activities/a7-problems/problem81012archive.shtml).
she must solve at least 16 of the first 22 problems correctly. This would give her a score of 100.5.
Difficulty: Medium-easy NCTM Standard: Problem Solving Standard: solve problems that arise in mathematics and in other contexts. Classification: Calculus and Analysis > Inequalities Number Theory > Arithmetic > Addition and Subtraction
x + 2. So x + 2 = 3x, from which x = 1. Thus the two integers are 1 and 3, and their sum is 4. Solution Answer (A):
Difficulty: Easy + 2. So x Algebra +2 = 3 x, from which x and = 1analyze . Thusmathematical the two integers are 1 NCTMxStandard: Standard: represent situations and structures symbols. 3,using and algebraic their sum is 4. Classification: Number Theory > Integers > Odd number
15
AMC 10 Student Practice Questions continued
The Dunbar family consists of a mother, a father, and some children. The average age of the members of the family is 20, the father is 48 years old, and the average age of the mother and children is 16. How many children are in the family?
Solution Answer (D): Sarah will receive 4.5 points for the three questions she leaves unanswered, so she must earn at least 100 − 4.5 = 95.5 points on the first 22 problems. Because 15 < 95.5 < 16, 6
(A) 2 (B) 3 (C) 4 (D) 5 (E) 6
2007 AMC 10 A, Problem #10— “Set up equations with two variables, and use the conditions above to eliminate one. ”
Solution Answer (E): Let N represent the number of children in the family and T represent the sum of the ages of all the family members. The average age of the members of the family is 20, and the average age of the members when the 48-year-old father is not included is 16, so 20 = This implies that 20N + 40 = T so 20N + 40 = 16N + 64. Hence 4N = 24 and N = 6.
(A) 13 (B) 14 (C) 15 (D) 16 (E) 17
2007 AMC 10 B, Problem #6— 2007 AMC 12 B, Problem #5— “Sarah must earn at least 95.5 points on the first 22 problems.”
XIV. Classroom Accessories
相关文档
最新文档